MODULE 6 (GREEN)

Ace your homework & exams now with Quizwiz!

The following statements regarding bacterial exotoxins are correct, except: A) They are integral parts of the cell wall. B) They are produced by both Staphylococcus aureus and Escherichia coli. C) They are polypeptides consisting of two functional regions, the one that binds to cell receptors and one that ha the toxic activity. D) Treatment of some exotoxins with formaldehyde yields a toxoid, which is used as the immunogen in certain vaccines.

A) They are integral parts of the cell wall.

A process of gene transfer where the carrier of DNA is a virus: A) Transduction B) Conjugation C) Transformation D) Both A and C

A) Transduction

A structurally mature infections viral particle: A) Virion B) Capsid C) Capsule D) Spore

A) Virion

The first drug available for HIV: A) ZDU B) RMP C) TMP D) CMC

A) ZDU

A chemical agent that kills the vegetative forms of pathogenic microorganisms but not necessarily the spores is known as: A) Antiseptic B) Disinfectant C) Sanitizer D) Germicide

B) Disinfectant

The first chemotherapeutic agent scientifically discovered and evaluated: A) Sulfonamide B) Penicillin C) Salvarsan D) None of the above

C) Salvarsan

A cellular structure equated for drug resistance is the: A) Endospore B) Exospore C) Transposon D) Capsule

D) Capsule

The heat-stable lipopolysaccharide in the outer membrane of gram-negative cell wall that is toxic to the host is known as: A) Tetanospasmin B) Aflatoxin C) Enterotoxin D) Endotoxin

D) Endotoxin

It causes ringworm infection with whitish patches on human skin: A) Tinea capitis B) Tines corporis C) Trichophyton rubrum D) Malasseria furfur

D) Malasseria furfur

Organisms that can grow at body temperature: A) Thermophiles B) Psychrophiles C) Acidophiles ) Mesophiles

D) Mesophiles

The primary site of electron transport system in eukaryotes: A) Ribosomes B) Nucleus C) Cytoplasm D) Mitochondria

D) Mitochondria

Several viruses infect the intestinal tract as their initial site of infection. Which one of the following is least likely to do this? A) Hepatitis A virus B) Poliovirus C) Rotavirus D) Mumps virus

D) Mumps virus

A ____ is a specific amount produced according to a single manufacturing order during a single period of production. A) Batch B) Lot C) Component D) None of the above

A

A ______ is a definite working rule regarding N, n and c. A) Sampling plan B) QC analysis C) Risk D) Chart

A

A chelate must always contain a/an: A) Multivalent metal B) Ethylenediamine group C) Amine group D) Triple bond

A

A concentration expression referring to the number of equivalents of solute per liter of solution is: A) Normality B) Molarity C) Molality D) Formality

A

A person who determines the presence of pyrogens and microorganisms of a pharmaceutical product is performing: A) Special method B) Identification test C) Pharmaceutical assaying D) Pharmacopoeial test

A

A sample of 0.1350 g of As2O3 was assayed iodimetrically using 23.4 mL of 0.1055N Iodine solution. The percentage purity of the sample is: At.wt. of As = 74.92; O = 16 A) 90.44 B) 90.23 C) 89.00 D) 90.54

A

A sample of 0.1350g of arsenic trioxide (As2O3) was assayed iodimetrically using 23.4mL of 0.1055N iodine solution. The percentage purity of the sample is: A) 90.44 B) 89.10 C) 90.54 D) 90.23

A

A sample of Magnesia Magma weighing 12.35g was dissolved in 50mL of 1.0340N sulfuric acid and titrated until end point is reached consuming 24.6mL of 1.1255N sodium hydroxide solution. The percentage of MgO content is: At.Wt. of Mg = 24.3: O = 16 A) 3.92 B) 4.0 C) 3.85 D) 3.91

A

A similar preparation in spectrometry refers to: A) Reference standard and sample identically made C) Sample greater than reference standard B) Reference standard greater than sample D) All of the above

A

A solution of known concentration used to standardize another solution is: A) Primary standard B) Dilute solution C) Secondary standard D) B ans C

A

A spectrophotometer differs from a colorimeter because it consists of: A) Prism B) Lamp house C) Cell compartment D) Optical scale

A

Acetylene and aldehyde group present in a compound that absorbs radiant energy are called: A) Chromophore B) Inactive C) Inert D) B and C

A

All of the following are optical devices or instruments, except: A) pH meter B) Spectrophotometer C) Colorimeter D) Flame photometer

A

An accurately measured sample of hydrogen peroxide 2.00g was dissolved in a mixture of 20mL water and 20mL and 20mL diluted sulfuric acid and was then titrated with 0.100N potassium permanganate consuming 30.0mL to reach the end point. Compute for the percentage of peroxide (MW = 34) A) 2.55% B) 5.1% C) 2.5% D) None of the above

A

An accurately measured sample of hydrogen peroxide 2g was dissolved in a mixture of 20mL water and 20mL diluted sulfuric acid and was titrated with 0.1N potassium permanganate consuming 30mL to reach the endpoint. Compute for the percentage of hydrogen peroxide (MW=34): A) 2.55% B) 5.1% C) 2.5% D) 5.5%

A

An instrument in spectrometry using the filter as radiant energy device: A) Filter photometer B) Infrared spectrometer C) UV - Vis spectrometer D) Flame photometer

A

Calculate the calcium carbonate content of a sample of chalk weighing 0.2545g and consuming 16.7 mL of 0.1150 M EDTA. At. Wt. Of Ca = 40.08 A) 75.52% B) 75.50% C) 37.76% D) 37.78%

A

Calculate the normality of sulfuric acid solution using 22.15 mL of it. The volume of NaOh used in the standardization was 20.75 mL and its normality was 1.1055. A) 1.0356 B) 1.3056 C) 1.0035 D) 1.0030

A

Calculate the normality of sulfuric acid solution using 22.5mL. The volume of standardized NaOH used in the standardization was 20.70mL, and its normality was 1.1055: A) 1.0171 B) 1.1071 C) 1.0175 D) 1.0711

A

Calculate the water content of an antibiotic powder using 350mg sample. The water equivalence factor (F) of the Karl Fischer reagent is 4.6 and the volume of the reagent used was 9.2 mL. The % of water content is: A) 12.09% B) 1.21% C) 12.11% D) A and B

A

Cause of quality variation due to men: A) Fatigue due to overwork B) Requires high salary C) Resourceful at work D) B and C

A

Complex inorganic compounds that are used to determine the end point in titration: A) Indicators B) Standard solutions C) Precipitating agents D) None of the above

A

Compute for the angle of repose of the sample powder using fixed funnel method if the height and diameter obtained was 5 cm and 13 cm respectively. A) 37.57o B) 36.57o C) 35.75o D) 36o

A

Compute for the iodine value of corn oil 0.2048g dissolved in 10 mL chloroform and 25 mL iodobromide TS was added, mixed and allowed to stand for one hour. Thirty mL of KI TS and 100 mL distilled water were added and titrated with 10 mL of 0.1065 N sodium thiosulfate. The volume of sodium thiosulfate consumed in the blank was 25.7. I is equal to 126.9. A) 103.61 B) 103.16 C) 101.36 D) 103.66

A

Constant weight in analytical procedures of drying means that consecutive weighing after heating and cooling do not differ by: A) More than 0.25mg B) More than 0.50mg C) More than 0.255mg D) Not more than 0.75mg

A

Content uniformity is to be performed for all tablets whose active ingredients are: A) 50mg or less B) 100mg or less C) 50mg or more D) None of the above

A

Content uniformity test is conducted in: A) 10 tablets B) 15 tablets C) 20 tablets D) 30 tablets

A

Content uniformity test is to be performed for all products whose active ingredient is: A) 50 mg or less B) 130 - 134 mg C) 130 mg or less D) 324 mg or more

A

Drug substances that are withheld from distribution until batch certification from BFAD has been received: A) Antibiotics B) Antineoplastics C) Vitamins D) All of the above

A

Efficiency of HEPA filters is assessed using: A) DOP test B) Water attack test C) Carr's index D) Bubble point test

A

Fats are: A) Esters of glycerin and fatty acid B) Polypeptides C) Compounds of glycerol, fatty acid. Phosphoric acid and choline D) Steroids with one or more hydroxyl groups E) Esters of fatty acids with high molecular weight alcohol

A

Find the acid value of a sample weighing 1.230 g that consumes 30.0 mL pf 0.110 N NaOH to bring about the end point using KOH in the titration: A) 150.54 mg/g B) 157.72 mg/g C) 109.18 mg/g D) 124.41 mg/g

A

Gas chromatography is used in the separation of: A) Volatile liquids B) Amino acid C) Carbohydrates D) Lipids E) Carboxylic acid

A

How many mL of water is expected from a 20 g Digitalis sample containing 1.5% moisture? A) 0.3 mL B) 30 mL C) 3 mL D) None of the above

A

How many samples should be tested if a batch consists of 120 bottles of paracetamol suspension when the square root method is used? A) 12 B) 11 C) 10 D) 9

A

If a 0.2250g of sodium bicarbonate (96.5% NaHCO3) is titrated with 0.1165N sulfuric acid. What volume of the acid is required to reach the end point? At.Wt. of Na = 23; H = 1.0; C = 12; O = 16 A) 22.19mL B) 21.98mL C) 21.20mL D) 21.22mL

A

If a 0.3800 g sample of sodium sulfate yielded 0.55 g of barium sulfate precipitate by gravimetric assay, what would be the percentage purity of sodium sulfate? At.wt.: Ba = 173.3; S = 32.06; Na = 23; O = 16 A) 88.11 B) 99.94 C) 99.15 D) 88.51

A

If a 0.4600g of potassium iodide (KI) yielded a 0.715g of silver iodide precipitate by gravimetric assay, what would be the percentage purity of KI? At.wt.: Ag = 107.87; K = 39.1; I = 126/9 A) 110.90 B) 110.89 C) 45.08 D) 109.90

A

In column chromatography, the resulting patterns with several bands is called: A) Chromatogram B) Eluate C) Spectrum D) Band E) A and C

A

In gas chromatography, the result of the assay is expressed in: A) Retention on the column B) Rf value C) Volume of gas used D) Adsorbent E) A and C

A

In microbial assay, the instrumental method used to prepare the inoculum is: A) Turbidimetry B) Nephelometry C) Chromatography D) A and B

A

In phenol content determination of a volatile oil, the layer in the graduated neck of the Cassia flask reached 2.7mL obtained from a sample of 10mL after treatment with KOH solution. The percentage of oil is: A) 73% B) 69% C) 7.3% D) 27%

A

In the assay of ascorbic acid by iodimetry, it acts as: A) Reducing agent B) Oxidizing agent C) Buffer D) Neutral agent

A

In the microbial assay of antibiotics, the microorganism used for Penicillin G is: A) Staphylococcus aureus C) Escherichia coli B) Pseudomonas aeruginosa D) Bacillus subtilis

A

Inspection stations are placed in the following areas, except: A) Analytical laboratory B) Manufacturing area C) Warehouse D) Packaging area

A

Iodometry is an indirect analysis of: A) Oxidizing agents B) Reducing agents C) Acids D) Bases

A

Limulus ameobocyte lysate (LAL) assay method is applicable in the determination of: A) Pyrogens B) Chemical impurities C) Microorganisms D) All of the above

A

Malic acid present in cherry juice may be determined by: A) Indirect permanganate oxidation method C) Alkalimetry B) Direct permanganate oxidation method D) Acidimetry

A

Most of the official drugs containing calcium and zinc are assayed by: A) EDTA method B) Gravimetry C) Nonaqueous titrimetry D) Acidimetry

A

Nephelometry is based on the measurement of light that is: A) Refelected B) Absorbed C) Transmitted D) Adsorbed by the particles of a suspension

A

One twentieth molar solution of sulfuric acid is: A) 0.05 M B) 0.1 M C) 0.005 M D) 0.5 M

A

Radiant energy required in the analysis of drugs under Fluorometry is in: A) UV region B) Visible region C) IR region D) A and B

A

Safety and toxicity tests are conducted for: A) Plastic tubings B) Solutions in vial C) Tablets D0 Oral liquids

A

Silica from the soil that is left after treatment of the ash with diluted HCl is the: A) Acid-insoluble ash B) Sulfated ash C) Residue on ignition D) Loss on ignition

A

Stoichiometric point in titration means that: A) Equivalent amounts of the titrant and analyte have reacted. B) Normality or molarity of the reactants are the same. C) End point in titration has been reached D) A and C

A

The First-In First-Out Policy must always be observed to: A) Assures that the oldest stock is used first B) Prevents contamination and mix-ups of materials C) Contains the information regarding the activity of the active ingredient D) All of the above

A

The QC manager is in charge of the following sections, except: A) Specification and assay development C) Research and development studies B) Analytical testing laboratory D) Biological testing

A

The assay procedures in volumetric precipitation were introduced by the following scientists, except: A) Arrhenius B) Mohr C) Fajan D) Volhard

A

The bisulfite method is used in the assay of volatile oils to determine: A) Aldehyde content B) Acid content C) Ester content D) Sulfur content

A

The calcium and sodium content of the blood can be determined by: A) Flame spectrometry B) Nephelometry C) Colorimetry D) Turbidimetry

A

The concentration of official diluted alcohol is: A) 10% B) 50% C) 80% D) 70%

A

The crude fiber of a drug usually consist of: A) Cellulose B) Cholesterol C) Phytosterol D) None of the above

A

The experimental way of determining when equivalent amounts of reactants have reacted together is: A) Titration B) Stoichiometric point C) Standardization D) Equivalence point

A

The following data were obtained after the moisture and total ash content determination of acacia powder: Wt. of empty crucible 52.452 g Wt. of crucible & sample 61.648 g Wt. of crucible & sample after drying to constant wt. 60.502 g Wt. of crucible and residue left after incineration 53.006g Moisture content in grams: A) 1.146 B) 1.144 C) 1.164 D) 1.166

A

The instrument used in measuring the optical activity of a sample: A) Polarimeter B) Refractometer C) Spectrometer D) Flame photometer

A

The masking agent used in the assay of Mg with EDTA in the presence of Al is: A) Triethanolamine B) Thioglycol C) Potassium cyanide D) Ammonium fluoride

A

The material used to make the cell holder for infrared analysis are, except: A) Sodium bromide B) Potassium bromide C) Sodium chloride D) Potassium chloride

A

The measurement of a base of a given sample by titration with a standard acid is: A) Acidimetry B) Compleximetry C) Alkalimetry D) Redox

A

The measurement of a weak base in nonaqueous medium of a given sample by titration with standard perchloric acid: A) Acidimetry B) Alkalimetry C) Compleximetry D) Redoc titration

A

The most effective chemical substance in removing stains of potassium permanganate: A) Oxalic acid B) Sodium thiosulfate C) Chalk D) Vinegar

A

The oxidation number of atomic iodine is: A) +1 B) +2 C) +4 D) +3

A

The period of stability of a preparation is the time from the date of manufacture of the formulation until its chemical or biological activity is not less than ___ of labeled potency: A) 90% B) 95% C) 93% D) 99%

A

The pharmaceutical excipient, which affects the disintegration of a tablet is: A) Disintegrant B) Lubricant C) Filler D) Colorant

A

The process by which the exact concentration of a solution is determined is: A) Standardization B) Neutralization C) Redox S) Acidimetry

A

The process of removing an appropriate number of items from a population in order to make interferences to the entire population is called: A) Sampling B) Inspection C) Statistic D) None of the above

A

The purity and strength of chemicals and drugs official in the USP/NF is usually expressed in terms of: A) Percentage B) g/mL C) mg/mL D) g/L

A

The purpose of using a combination of indicators is: A) To attain a sharp end point C) To attain a complete reaction B) To facilitate reaction D) A and B

A

The recommended maximum limit for overage of vitamins is: A) 30% B) 10% C) 90% D) None of the above

A

The residue after incineration of a drug was 0.1185 g from a sample of 7.0 g. The percentage of ash is: A) 1.56 B) 2.56 C) 2.0 D) 1.55

A

The soluble constituents of crude drug in a certain solvent: A) Extractive B) Ash C) Crude fiber D) Phytosterol

A

The study of the optical activity of a substance is: A) Polarimetry B) Refractometry C) Spectrophotometry D) Potentiometry

A

The terms used when equivalent amounts of each reactant have reacted are the following, except: A) End point B) Equivalence point C) Stoichiometric pont D) Theoretical point

A

The titer value of 1.0mL of 0.1N ceric sulfate for arsenic trioxide (MW = 197.94) A) 9.496 mg B) 197.84 mg C) 9.892 mg D) 14.838 mg

A

The unsaponified matter present in animal fat is: A) Cholesterol B) Phytosterol C) Lard D) Wax

A

This is the method of analysis based on the measurement of current resulting from the electrolysis of an electroactive species at a given electrode potential under controlled condition: A) Potentiometry B) Polarimetry C) Polarography D) Refractometry E) pH measurement

A

This is the sum of all tests performed to determine the conformance of the product to specifications: A) Quality control B) Quality Assurance C) CGMP D) A and B

A

To assure stability of the product until expiry date, the label should indicate: A) Proper storage B) Proper dosage C) Proper color D) Proper drug administration

A

To control finished products in the warehouse of a company, the distribution practice is: A) First-in first-out B) Samples needed C) Ratio and proportion D) A and B

A

To find the mEq factor of an oxidizing agent, divide the molecular weight by: A) No. of electrons gained B) No. of electrons lost C) Valence D) Replaceable hydrogen

A

To identify the place where materials have to be stored in a material warehouse, a _____ is pasted on the container of the material. A) Sticker B) Label C) Ribbon D) None of the above

A

Trace minerals in a multivitamin preparation can be assayed faster by: A) Atomic Absorption Spectrophotometer C) Gas Liquid Chromatography B) UV-Vis Spectrophotometer D) High Pressure Liquid Chromatography

A

Turbidimetric and nephelometric methods of assay are applied to: A) Suspensions B) Solutions C) Colored samples D) A and B

A

Twenty tablets of ascorbic acid weighed 4.2500g and a powdered sample of 0.3075g was titrated consuming 21.5mL of 0.1085N Iodine solution. Each mL of 0.1N Iodine is equivalent to 8.80mg of ascorbic acid. What is the amount of ascorbic acid per dose of two tablets? A) 283.72 mg B) 141.86 mg C) 283.22 mg D) A and C

A

USP requires not less than 2.5g and not more than 3.5g of hydrogen peroxide /100 parts. Does the sample conforms with the standard requirement? A) Yes B) No C) Maybe D) Cannot be determined

A

Using a sampling plan by military standard 105 D, the sample size required is obtained from: A) Master table B) Ratio and proportion C) Samples needed D) A and B

A

What would be the aldehyde content of peppermint oil if the sample used was 15mL and the insoluble portion as read in the graduated neck of the Cassia flask was 2.2mL. The percentage of aldehyde content is: A) 85.33 B) 83.33 C) 84.30 D) 14.67

A

When a strong acid is titrated against a weak base, the indicator is: A) Methyl red B) Phenolphthalein C) Methyl orange D) Crystal violet

A

Which of the following is a reducing agent? A) Ascorbic acid B) KMNO4 C) Iodine D) Hydrochloric acid

A

Which of the following substances is assayed by residual alkalimetric analysis? A) Aspirin B) Sodium hydroxide C) Hydrochloric acid D) Zinc oxide

A

Influenza develops rapidly and lasts only for a short time. This is described as: A) Acute B) Subacute C) Chronic D) Latent

A) Acute

Unicellular prokaryotic organism that divides by binary fission: A) Bacteria B) Fungi C) Protozoa D) Helminths

A) Bacteria

Deficiency of thiamine leads to: A) Beri-beri B) Pellagra C) Stomatitis D) A and B E) B and C

A) Beri-beri

Food processing technique that protects food from oxidative deterioration and growth of aerobic microorganism: A) Canning B) Pasteurization C) Irradiation D) Drying E) None of the above

A) Canning

The posterior part of this parasite has a twisted appearance: A) Chilomastix mesnili B) Giardia lamblia C) Trichomonas hominis D) Retortamonas

A) Chilomastix mesnili

The causative agent of gas gangrene. A) Clostridium perfringens D) Corynebacterium diphtheriae B) Clostridium tetani E) Clostridium dificile C) Clostridium botulinum

A) Clostridium perfringens

The following organisms are recognized causes of diarrhea, except: A) Clostridium perfringens B) Streptococcus fecalis C) Escherichia coli D) Vibrio cholerae

A) Clostridium perfringens

A trace mineral necessary for heme synthesis, electron transport and wound healing: A) Copper B) Manganese C) Chromium D) Zinc E) None

A) Copper

The following viruses possess double-stranded DNA as its genome, except: A) Coxsackie virus B) Herpes simplex virus C) Rotavirus D) Adenovirus

A) Coxsackie virus

Herpes simplex virus A) DNA enveloped virus D) RNA non-enveloped virus B) DNA non-enveloped virus E) Viriod C) RNA enveloped virus

A) DNA enveloped virus

Its function is mainly to serve as an advisory body to the local executive or local legislative on health related matters: A) DOH B) WHO C) QUERT D) Sentrong Sigla E) None of the above

A) DOH

The "four o'clock habit" promoted by the Department of Health is designed to prevent the spread of: A) Dengue fever B) Tuberculosis C) Malnutrition D) Malaria

A) Dengue fever

An algae group responsible for red tide poisoning: A) Dinoflagellate B) Diatom C) Euglenoid D) None of the above

A) Dinoflagellate

Inhibits protein synthesis by blocking elongation factor-2 A) Diphtheria toxin D) Toxic shock syndrome toxin B) Tetanus toxin E) Cholera toxin C) Botulinum toxin

A) Diphtheria toxin

Trophozoite with nucleus seen without difficulty in fresh, unstained preparation: A) Entamoeba coli B) Endolimax nana C) Entamoeba histolytica D) Iodamoeba butschlii E) Dientamoeba fragilis

A) Entamoeba coli

The eggs of this organism are most likely to be found through anal swabs or cellophane tape: A) Enterobius vermicularis B) Trichuris trichiura C) Ascaris lumbricoides D) Trichinella spiralis

A) Enterobius vermicularis

Dermatophytes that infect special keratinized areas of the body, skin and nails only are likely to belong to genus: A) Epidermophyton B) Trichophyton C) Microsporum D) Trichosporum

A) Epidermophyton

The bacterium that produces a toxin that activates adenylate cyclase, resulting in accumulation of cyclic AMP in the epithelial cells of the mucosal lining, is: A) Escherichia coli B) Pseudomonas aeruginosa C) Streptococcus pneumoniae D) Staphylococcus aureus

A) Escherichia coli

This chemical agent is used extensively in hospitals for the sterilization of heat sensitive materials. It kills by alkylating both proteins and nucleic acids: A) Ethylene oxide B) Formaldehyde C) Acids and alkalis D) Hydrogen peroxide

A) Ethylene oxide

The bacterial organelle that is used for motility: A) Flagella B) Pili C) Cilia D) Pseudopods

A) Flagella

Donated blood containing antibody to this RNA virus ahould not be used for transfusion. A) Hepatitis C virus D) Dengue virus B) Cytomegalovirus E) St. Louis encephalitis virus C) Human papilloma virus

A) Hepatitis C virus

The causative agent of herpes labialis (fever blisters or cold sores), keratitis and encephalitis. A) Herpes Simplex virus Type 1 D) Cytomegalovirus B) Varicella-Zoster virus E) Epstein-Barr virus C) Herpes simplex virus Type 2

A) Herpes Simplex virus Type 1

Bacteria that derive their energy from oxidation of organic molecules are known as: A) Heterotrophs B) Autotrophs C) Chemoautotrophs D) Photoautotrophs

A) Heterotrophs

Nosocomial infections are: A) Infections developed while the patient is in the hospital B) Infections of the nasal area C) Infections where pathogens enters the body through the nose D) Infections among animals

A) Infections developed while the patient is in the hospital

The ability of a microorganism to enter a host, grow, reproduce and spread throughout its body is known as: A) Invasiveness B) Pathogenicity C) Toxigenicity D) Virulence

A) Invasiveness

The two major components of virulence include: A) Invasiveness and toxigenicity B) Intracellular and extracellular survival C) Adherence factors and antiphagocytic mechanisms D) Capsules and spreading factors

A) Invasiveness and toxigenicity

Listeria monocytogenes has all of the following characteristics, except: A) It is a spore-former. B) It is a short, gram-positive rod C) It can live inside phagocytic cells. D) It is actively motile

A) It is a spore-former.

A gram negative, non-motile rod that produces a large mucoid colony on agar medium was isolated from a chronically ill patient who died of pneumonia. This organism probably belongs to the genus: A) Klebsiella B) Pseudomonas C) Proteus D) Serratia

A) Klebsiella

Lesions in the oral cavity caused by measles virus are known as: A) Koplik spot B) Peyer's pathches C) Rose spots D) Rashes

A) Koplik spot

This causes visceral leishmaniasis: A) Leishmania donovani C) Leishmania braziliensis B) Leishmania tropica D) Trypanosoma cruzi

A) Leishmania donovani

The most important anti-phagocytic structure of the Group A streptococci: A) M protein B) Group A carbohydrate C) Hyaluronic acid capsule D) Protein A

A) M protein

A gram negative, kidney bean shaped diplococci, oxidase positive with large polysaccharide capsule referred to as glycocalyx. A) Meningococcus D) Clostridium tetani B) Gonococcus E) Listeria monocytogenes C) Bacillus antracis

A) Meningococcus

A rod-like bacterium that grows optimally at mammalian body temperature and is killed by the presence of oxygen is best described as: A) Mesophilic, obligate anaerobic bacilli B) Mesophilic, obligate aerobic bacilli C) Psychrophilic, facultative anaerobic streptobacillus D) Psychrophilic, obligate anaerobic bacilli

A) Mesophilic, obligate anaerobic bacilli

An index that measures the number of individuals who have become ill because of a specific disease within a susceptible population during a specific period: A) Morbidity rate B) Prevalence rate C) Mortality rate D) All of the above

A) Morbidity rate

32. The following are zoonotic diseases, except: A) Mumps B) Leptospirosis C) Anthrax D) Brucellosis

A) Mumps

An acid-fast bacillus that have mycolic acid in its cell wall, which can be seen in a palisade arrangement. A) Mycobacterium leprae D) Treponema pallidum B) Mycobactrium tuberculosis E) Leptospira interrogans C) Borrelia burgdorferi

A) Mycobacterium leprae

Hansen's disease is caused by: A) Mycobacterium leprae B) Mycobacterium tuberculosis C) Myoplasma leprae D) None of the above

A) Mycobacterium leprae

The organism with atypical cell walls: A) Mycoplasmas B) Rickettias C) Chlamydias D) Viruses

A) Mycoplasmas

The following can be sterilized in an autoclave, except: A) Olive oil B) Culture media C) Water D) Glassware

A) Olive oil

The sudden unexpected occurrence of a disease in a given population: A) Outbreak B) Zoonotic C) Endemic D) Sporadic

A) Outbreak

Which of the following forms of immunity to viruses would be least likely to be lifelong? A) Passive immunity B) Passive-active immunity C) Active immunity D) Cell-mediated immunity

A) Passive immunity

Eosinophilic stippling of erythrocytes (Schauffer.s dots) is often seen in infections with: A) Plasmodium vivax B) Plasmodium malariae C) Plasmodium falciparum D) Plasmodium ovale

A) Plasmodium vivax

The malarial parasite that enlarges red blood cells: A) Plasmodium vivax B) Plasmodium malariae C) Plasmodium falciparum D) Plasmodium ovale

A) Plasmodium vivax

This organism has lemon-shaped cysts: A) Plasmodium vivax B) Plasmodium malariae C) Plasmodium falciparum D) Plasmodium ovale

A) Plasmodium vivax

It refers to water suitable for drinking: A) Potable B) Edible C) Bacteria-free D) Odor-free

A) Potable

This is a period between conceptions through complete delivery of the product of conception: A) Pregnancy B) Fertilization C) Implantation D) Cleavage E) None

A) Pregnancy

A gram-negative rod was isolated from the urine of a patient suspected of having a UTI. The organism is lactose negative, urease positive and motile. It most likely belongs to the genus: A) Proteus B) Escherichia C) Serratia D) Klebsiella

A) Proteus

It describes the amounts of energy, protein, minerals and vitamins needed by normal healthy individual: A) RDA B) REA C) RAD D) RAE E) None

A) RDA

This causes Egyptian hematuria: A) Schistosoma hematobium C) Schistosoma japonicum B) Schistosoma mansoni D) Fasciola hepatica

A) Schistosoma hematobium

The sexual cycle is the: A) Sporogony B) Schizogony C) Either B or C D) Neither B nor C

A) Sporogony

The causative agent of food poisoning, TSS and skin boils. A) Staphylococcus aureus B) Streptococcus pyogenes C) Streptococcus pneumoniae D) Staphylococcus epidermidis E) Streptococcus agalactiae

A) Staphylococcus aureus

Lymphocytes that have a high affinity for HIV: A) T-helper lymphocytes B) B-lymphocytes C) Phagocytes D) T-cytotoxic lymphocytes

A) T-helper lymphocytes

In clinical cases of malaria, the most favorable time to find parasites in the blood is: A) The period just before a paroxysm B) At the beginning of a paroxysm C) During the late paroxysmal stage D) The period just following a paroxysm

A) The period just before a paroxysm

Koch's postulates include the following, except: A) The suspected organism should be present in health individuals B) The organism must be isolated and grown as pure culture in the laboratory C) The organism must initiate the symptoms of the disease in healthy animals D) The organism must be re-isolated from the animal and be cultured again in the laboratory

A) The suspected organism should be present in health individuals

The following statements concerning human rabies vaccine are correct, except: A) The vaccine contains live, attenuated rabies virus. B) If the patient was bitten by a wild animal such as skunk, the rabies vaccine must be given. C) When the vaccine is used for post-exposure prophylaxis, rabies immune globulin must also be given. D) The virus in the vaccine is grown in human cell cultures thus decreasing the risk of allergic encephalomyelitis.

A) The vaccine contains live, attenuated rabies virus.

A theory that states that life originates from non-life: A) Theory of Spontaneous Generation C) Germ Theory B) Koch's Postulates D) None of the above

A) Theory of Spontaneous Generation

This statement is true about viral infection: A) Viral infections are self-limiting B) Viral infections confer lifetime immunity C) Viral infections are treated by antibiotics D) All statements are correct

A) Viral infections are self-limiting

The following statements about lysogeny are correct, except: A) Viruses replicate independently of bacterial genes. B) Viral genes responsible for lysis are repressed. C) Viral DNA is integrated into bacterial DNA. D) Some lysogenic bactriophage encode toxins that cause human diseases.

A) Viruses replicate independently of bacterial genes.

An example of ionizing radiation: A) X-ray B) UV C) Sunlight D) All of the above

A) X-ray

Urinary tract infection (UTI), sepsis neonatal meningitis and "travelers diarrhea" are the most common diseases caused by this gram negative, non-lactose fermenting bacilli. A. Escherichia coli B. Salmonella typhi C. Shigella dysenteriae D. Vibrio cholerae E. Camphylobacter jejuni

A. Escherichia coli

A buret with a glass stopcock can be used for: A) Alcohols B) Acids C) Bases D) Salts

B

A combining molecule containing one or more groups that donate electrons will form ____ with a metal. A) Complex B) Chelate C) Sequestering agent D) Masking agent

B

A plot of absorbance against concentration of a standard drawn in straight line is: A) Charle's B) Beer's C) Lambert's D) B and C

B

A term in compleximetry used to indicate a determination of a metal in the presence of another metal is: A) Redox B) Masking C) Linking D) Complexing

B

A type of titration where the first titrant is added in excees with the sample and the second titrant reacts with the added excess is: A) Back B) Direct C) Residual D) A and C

B

Amount of dissolved drug can be determined by: A) Analytical balance B) Dissolution tester C) Brookefield viscometer D) DOP spray

B

An sample of sodium sulfate weighing 1.800g yielded 0.900g of barium sulfate (MW = 233.25). Calculate the percentage of sulfur ( AW = 32.06) in the sample. A) 12.18% B) 6.870% C) 27.48% D) 30.44%

B

Analysis wherein the constituents of a sample are separated and then the product is weighed: A) Volumetric B) Gravimetric C) Special method D) Gasometric

B

Assay of atropine in Belladonna is an example of: A) Proximate assay B) Ultimate assay C) Both A and B D) Neither A nor B

B

Calculate the calcium carbonate content of a sample of chalk weighing 0.2545g and consuming 16.67mL of 0.1150M of EDTA solution in titration. At.Wt of Ca = 40.08; C = 12; O = 16. The percentage of CaCO3 is: A) 75.52 B) 75.50 C) 37.76 D) 37.78

B

Calculate the water content of streptomycin powder weighing 4.20g as sample. The water equivalence factor (F) of the Karl Fischer reagent was found to be 5.1 and the volume consumed was 11.50. The % of water is: A) 1.39 B) 1.40 C) 1.41 D) 13.96

B

Ceric sulfate is a/an: A) Precipitating agent B) Oxidizing agent C) Reducing agent D) Complexing agent

B

Compute for the Rf value of a substance if the substance traveled by the solvent is 12.5 cm and the distance traveled by the sample is 7.5 cm: A) 0.70 B) 0.60 C) 1.67 D) 1.66

B

Determination of iodine value of oils depends on: A)Substitution with iodine for the hydrogen in the fatty acid B)Addition of iodine at the double bond of the fatty acid C)Oxidation of the fatty acid by iodide D)All of the above

B

During an in-process control testing, the following data were obtained from loss on drying test: Wt. of sample before drying 1.146 g Wt. of tare & sample after drying 25.653 g Wt. of tare 24.632 g What is the weight of moisture lost? A) 1.021 g B) 0.125 g C) 1.125 g D) 0.021 g

B

Expression of concentration in volumetric solutions, except: A) Normality B) Titer C) Molarity D) Molality

B

Hexane is the best solvent for extracting: A) Resins B) Fats C) Volatile oils D) Acids

B

If 1mL of HCl is equivalent to 1.5mL of 0.5N NaOH, the volume of the alkali that would be consumed by 30mL of acid is: A) 4.5mL B) 45mL C) 20mL D) 30mL

B

If a sample is beeswax is found to have an acid number of 15.5 and a saponification value of 71.2, the ester value of the sample is: A) 86.7 B) 55.7 C) 90 D) None of the above

B

In IR spectrometry, the finger print region is: A) Near IR B) Medium IR C) Far IR D) A and B

B

In gravimetric analysis, when the process used in extraction to obtain the original constituent, this method belongs to: A) Chemical B) Physical C) Precipitation D) Any of the above

B

In spectrophotometry procedure for assay, aside from the sample or unknown, this is also required: A) Raw material of the sample B) Reference C) Dosage form D) A and C

B

In the assay of ZnO (MW = 81.38), each mL of 1N of H2SO4 is equivalent to ____ of ZnO. A) 81.38 mg B) 40.69 mg C) 27.13 mg D) 20.34 mg

B

In the standardization of HCl using pure anhydrous sodium carbonate as primary standard and methyl orange as indicator, 1.0mL HCl was found to be equivalent to 0.05g of sodium carbonate (MW = 106). The normality of HCl is: A) 1.0 N B) 0.94 N C) 0.4716 N D) None of the above

B

Materials to be sampled include the following, except: A) Final products B) Records C) Intermediate products D) Raw materials

B

Neutralization reactions in titrimetric methods of analysis may be: A) Precipitimetry B) Alkalimetry C) Compleximetry D) A and C

B

One batch can be broken into several: A) Ingredients B) Lots C) Components D) None of the above

B

One of the fundamental laws in spectrophotometry is: A) Law of Mass Action B) Beer's Law C) Boyle's Law D) Newton's law

B

Potency of a 100mg tablet at the expiry date: A) 95mg B) 90mg C) 105mg D) 110mg

B

Potentiometry finds application in: A) Biologic assay B) pH determination C) Qualitative analysis D) A and B

B

Residual method titration with EDTA is applicable to what metal ion: A) Calcium B) Aluminum C) Zinc D) Magnesium

B

The Karl Fischer reagent used for moisture content determination consists of the following except: A) Pyridine B) Acetone C) Sulfur dioxide D) Iodine

B

The acid necessary for permanganate titrations is: A) Acetic acid B) Sulfuric acid C) Hydrochloric acid D) Nitric acid

B

The apparatus used to keep samples free from moisture is: A) Oven B) Dessicator C) Furnace D) A and C

B

The assay of the aldehyde content of volatile oils may be done by: A) Extraction B) Bisulfite method C) Bobcock method D) Gravimetric method

B

The end point in iodimetry using starch TS as indicator is: A) Blue color B) Disappearance of blue color C) Greenish blue color D) A or C

B

The end point in the water content determination using Karl Fischer reagent is determined by: A) Use of chemical indicator C) Use of spectrometer B) Use of potentiometer or volumeter D) A or B

B

The following data were obtained after the moisture and total ash content determination of acacia powder: Wt. of empty crucible 52.452 g Wt. of crucible & sample 61.648 g Wt. of crucible & sample after drying to constant wt. 60.502 g Wt. of crucible and residue left after incineration 53.006g Percent moisture: A) 12.64% B) 12.46% C) 12.45% D) 12.54%

B

The following data were obtained after the moisture and total ash content determination of acacia powder: Wt. of empty crucible 52.452 g Wt. of crucible & sample 61.648 g Wt. of crucible & sample after drying to constant wt. 60.502 g Wt. of crucible and residue left after incineration 53.006g Percent total ash: A) 6.20% B) 6.02% C) 6.22% D) 6.92%

B

The indicator for EDTA direct titration of calcium carbonate: A) Thymol blue B) Hydroxynaphthol blue C) Methyl red D) Methylene blue

B

The method of assay for Vitamin B12 and calcium pantothenate is by: A) Iodimetry B) Microbial-turbidimetric method C) Fluorometry D) Redox titration

B

The molecular weight of NaOH is 40. How many grams of NaOH pellets are needed to make 500mL of 1.5N solution? A) 20 B) 30 C) 40 D) 60

B

The name of the originator who introduced the use of 0.1N silver nitrate as the titrant and potassium chromate TS as the indicator in volumetric precipitation of chlorides is: A) Volhard B) Mohr C) Fajan D) Beer

B

The official USP gravimetric methods are: 1. Assay involving direct ignition 2. Assay involving sublimation 3. Assay involving extraction 4. Assay involving precipitation followed by ignition A) 1, 3 and 4 B) 2, 3 and 4 C) 1, 2 and 4 D) 1, 2 and 3

B

The pH of a solution is usually measured by using: A) Platinum electrode B) Glass electrode C) Mercury electrode D) Calomel electrode

B

The reading that must be obtained in a spectrophotometer: A) Concentration error B) Absorbance C) Retention D) Angular rotation

B

The reagent that loses the electron/s in a redox reaction is the: A) Substance that decreases in oxidation number C) Reducing agent B) Oxidizing agent D) Substance reduced

B

The separation of the components of an organic dye mixture is accomplished by using: A) PC B) TLC C) HPLC D) GC E) Any of the above

B

The stationary phase used in gas chromatography is: A) Gas B) Liquid C) Substrate D) Cellulose E) Chromatogram

B

The temperature for ignition described as dull red heat is: A) 500 - 1000 oC B) 550 - 700 oC C) 500 - 750 oC D) 300 - 450 oC

B

The titrant most commonly used for direct compleximetry titration is: A) Zinc sulfate B) EDTA C) AgNO3 D) HclO4

B

The type of ash where sulfuric acid is used to whiten the ash is: A) Acid-insoluble ash B) Sulfated ash C) Loss on ignition D) Residue on ignition

B

This is the acceptable tablet hardness range of chewable tablets: A) 4 - 10 kg B) 2 - 3 kg C) 5 - 8 kg D) 2 - 8 kg

B

This is the primary cause of product instability and involves the addition of oxygen or the removal of hydrogen: A) Incompatibility B) Oxidation-Reducation C) Hydrolysis D) Racemization E) Decarboxylation

B

Volatile oils are complex compounds composed of the following, except: A) Phenols B) Fatty acids C) Hydrocarbons D) Aldehydes

B

Water attack test is used in this type of glass container: A) Type I B) Type II C) Type III D) Type III

B

What would be the accepted weight range of tablets weighing 130mg? A) 123.5 - 136.5 mg B) 120.25 - 139.75 mg C) 117 - 123 mg D) 120.9 - 139.1 mg

B

Which of the following is an oxidizing agent? A) Ascorbic acid B) KMNO4 C) Sodium thiosulfate D) Sodium nitrite

B

______ is the comparing of attributes and dimensions of a product against a standard to find out if the product is within the prescribed limit. A) Sampling B) Inspection C) Analysis D) Action

B

Ribosomes of Candida albicans are referred to as: A) 70s B) 80s C) 90s D) 100s

B) 80s

Dengue virus vector: A) Plasmodium falciparum B) Aedes aegypti C) Anopheles mosquito D) None of the above

B) Aedes aegypti

An immediate hypersensitivity reaction following exposure of a sensitized individual to the appropriate antigen: A) Hay fever B) Anaphylaxis C) Asthma D) Desensitization

B) Anaphylaxis

"Old world" hookworm: A) Necator americanus B) Ancylostoma duodenale C) Ancylostoma caninum D) Toxocara canis

B) Ancylostoma duodenale

Definitive host of human malarial parasites: A) Any domestic animal B) Anopheles mosquito C) Man D) B and C

B) Anopheles mosquito

A drug for treatment of alcoholism, which acts to diminish the pleasurable effects of alcohol: A) Naltrexone B) Antabuse C) Tranquilizers D) All E) None

B) Antabuse

The most important antibody that plays a role in the pathogenesis of systemic lupus erythematosus is: A) Antibody thyroglobuin B) Antibody to DNA C) Antibody to mitochondria D) Antibody to smooth muscle

B) Antibody to DNA

The Coomb's test is the most important laboratory method in the diagnosis of: A) Myasthenia gravis B) Autoimmune hemolytic anemia C) Waldenstrom's macroglubilemia D) Rheumatoid arthritis

B) Autoimmune hemolytic anemia

Plastics that can be decomposed by microorganism are said to be: A) Recyclable B) Biodegradable C) Earth-friendly D) Reversible

B) Biodegradable

Air contaminants causing death by asphyxiation in high concentration: A) Sulfur dioxide B) CO C) CO2 D) Lead E) None of the above

B) CO

The primary natural nutrients of bacteria are: A) Carbohydrates/Nucleic acids/Steroids B) Carbohydrates/Amino acids/Lipids C) Nucleic acids/Chitin/Peptidoglycans D) Lipids/Polysaccharides/Lactoferrin

B) Carbohydrates/Amino acids/Lipids

Man may acquire Toxoplasma from: A) Mice B) Cats C) Dogs D) Pigs

B) Cats

The following are true about moist heat sterilization, except: A) Makes use of an autoclave B) Causes oxidation of cell components C) Makes use of steam under pressure D) Can kill both vegetative cells and spores

B) Causes oxidation of cell components

Aplastic anemia is a rare but usually fatal side effect associated with the use of this antibiotic: A) Gentamicin B) Chloramphenicol C) Bacitracin D) Polymixin

B) Chloramphenicol

The disinfectant of choice for municipal water supplies: A) Lysol B) Chlorine C) Ozone D) Reverse osmosis

B) Chlorine

The presence of cryoglobulins in the serum of a patient may indicate that the patient has: A) Anemia B) Circulating immune complexes C) Hashimotos's thyroiditis D) Pernicious anemia

B) Circulating immune complexes

The bacterial toxin that attaches to the presynaptic terminals of cholinergic nerves, where it blocks the release of acetycholine, is produced by: A) Bacillus anthracis B) Clostridium botulinum C) Clostridium tetani D) Corynebacterium diphtheriae

B) Clostridium botulinum

Characterized physiologically by a "sardonic smile." A) Clostridium perfringens D) Corynebacterium diphtheriae B) Clostridium tetani E) Clostridium dificile C) Clostridium botulinum

B) Clostridium tetani

It is an important cause of pneumonia in immunocompromised patients. A) Hepatitis C virus D) Dengue virus B) Cytomegalovirus E) St. Louis encephalitis virus C) Human papilloma virus

B) Cytomegalovirus

The capsule of Bacillus anthracis is composed of: A) L-glutamic acid B) D-glutamic acid C) Polysaccharide D) Nucleic acid

B) D-glutamic acid

Human papillomavirus A) DNA enveloped virus D) RNA non-enveloped virus B) DNA non-enveloped virus E) Viriod C) RNA enveloped virus

B) DNA non-enveloped virus

It refers to the killing of many, but not all microorganisms: A) Sterilization B) Disinfection C) Antisepsis D) Asepsis

B) Disinfection

A hapten is a substance that: A) Induces cellular immune response but not antibody production B) Does not induce any immune response when given alone but does elicit an immune response when coupled to a larger molecule C) Induces tolerance when given alone D) When coupled to a larger molecule can be recognized by B lymphocytes but not T lymphocytes

B) Does not induce any immune response when given alone but does elicit an immune response when coupled to a larger molecule

Regarding the nature of medicinally important viruses, which one of the following statements is least accurate? A) Poliovirus is a non-enveloped virus with RNA as its genome. B) Epstein-Barr virus is a non-enveloped virus with RNA as its genome. C) Hepatitis B is an enveloped virus with RNA as its genome. D) Influenza virus is an enveloped virus with RNA as its genome.

B) Epstein-Barr virus is a non-enveloped virus with RNA as its genome

The following statements about the normal flora are correct, except: A) The most common organism found on the skin is Staphylococcus epidermidis. B) Escherichia coli is a prominent member of the normal flora of the throat. C) Colon is the major site where Bacteroides fragilis can be found. D) Nose is one of the most common sites where Staphylococcus aureus can be found.

B) Escherichia coli is a prominent member of the normal flora of the throat.

Phylogenetic classifications are based on: A) Shared morphologic attributes B) Evolutionary relationships C) Shape D) Unusual growth characteristics

B) Evolutionary relationships

Man is often infected by peeling water chestnuts with his teeth, which contain this organism: A) Fasciola hepatica B) Fasciolopsis buski C) Paragonimus weatermani D) Chlonorchis sinensis

B) Fasciolopsis buski

The Boracay water was declared unsafe due to: A) Industrial wastes B) Fecal coliforms C) Red tide D) Oil spills

B) Fecal coliforms

It is the method of choice in sterilizing heat sensitive solutions: A) Autoclave B) Filtration C) Radiation D) Pasteurization

B) Filtration

This mediates the attachment of bacteria to specific receptors on human cell surfaces, which is the necessary step in the initiation of infection for some organisms: A) Flagella B) Fimbriae C) Capsule D) Endospores

B) Fimbriae

An object that is able to harbor and transmit microorganisms: A) Mite B) Fomite C) Arthropod D) Vector

B) Fomite

The following are contributions of Louis Pasteur in microbiology, except: A) Terms "aerobic" and "anaerobic" C) Pasteur flask B) Fractional sterilization D) Fermentation

B) Fractional sterilization

An intracytoplasmic granule may contain: A) Nucleic cids B) Glycogen C) Steroids D) Spindle fibers

B) Glycogen

A gram negative, kidney bean shaped diplococci causing gonorrhea. A) Meningococcus D) Clostridium tetani B) Gonococcus E) Listeria monocytogenes C) Bacillus antracis

B) Gonococcus

The following pathogens are likely to establish chronic or latent infections, except: A) Cytomegalovirus B) Hepatitis A virus C) Hepatitis B virus D) Herpes simples virus

B) Hepatitis A virus

Bacteria that ferment substrates to single end products are called: A) Heterofermenters B) Homofermenters C) Glyoxalate shunt fermenters D) Oxidative fermenters

B) Homofermenters

Eggs in the 2-8 cell stage of development often appear in the stool in infections with A) Whipworm B) Hookworm C) Pinworm D) Large intestinal roundworm

B) Hookworm

The main host defense against bacterial exotoxins is: A) Activated macrophages secreting proteases B) IgG and IgM antibodies C) Helper-T cells D) Modulation of host cell receptors in response to the toxin

B) IgG and IgM antibodies

Artificially acquired passive immunity is developed after vaccination with: A) Attenuated microorganism C) Toxoids B) Immunoglobulin preparations D) Cyanobacteria

B) Immunoglobulin preparations

Toxoplasma is probably transmitted to man by: A) Bite of the vector insect B) Ingestion of a resistant oocyst C) In the body of the parasitic nematode D) Ingestion of infected food

B) Ingestion of a resistant oocyst

The use of nitrite as preservative for food is discouraged because: A) It can discolor the meat B) It can react with amines to form carcinogenic nitrosamines C) It decomposed to nitric acid which can react with heme pigments D) It is not readily available

B) It can react with amines to form carcinogenic nitrosamines

Which of the following statements best describe a viriod? A) It is a defective virus that is missing the DNA coding for the matrix protein. B) It consists of RNA without a protein or lipoprotein outer coat. C) It causes tumor in experimental animals. D) It requires RNA polymerase in the particle for replication to occur.

B) It consists of RNA without a protein or lipoprotein outer coat.

Which of the following statements is true about bacterial conjugation? A) It requires the F factor B) It is controlled by F plasmid and sex pili C) It is mediated by a bacterial virus D) It is found only in gram-negative bacteria

B) It is controlled by F plasmid and sex pili

Gram-negative bacterial cell wall contains the following components, except: A) Lipoproteins B) Lipotechoic acid C) Lipopolysacchatide D) Phospholipid

B) Lipotechoic acid

Stage of alcoholism wherein the person experiences blackouts: A) Initial stage B) Middle stage C) Third stage D) final stage E) None

B) Middle stage

An acid-fast, catalase negative bacillus that can be grown in Lowenstein-Jensen medium and is capable of producing Niacin. It can be treated with Rifampicin, Pyrazinamide and INH. A) Mycobacterium leprae D) Treponema pallidum B) Mycobactrium tuberculosis E) Leptospira interrogans C) Borrelia burgdorferi

B) Mycobactrium tuberculosis

The smallest known free-living microorganism is: A) Viruses B) Mycoplasma species C) Rickettsia D) Chlamydiae

B) Mycoplasma species

Another priority program of DOH, which emphasizes partnership and shared responsibility for health among various sectors: A) Health Sector Reform Agenda B) National Health Objectives C) Sentrong Sigla D) Health Passport Initiative E) None of the above

B) National Health Objectives

An outbreak of sepsis causes by Staphylococcus aureus has occured in the newborn nursery. You are called to investigate. According to your knowledge of the norma flora , what is the most likely source of the bacteria? A) Colon B) Nose C) Throat D) Vagina

B) Nose

The following viruses possess an outer envelope of lipoprotein, except: A) Varicell-zoster virus B) Papillomavirus C) Influenza virus D) HIV virus

B) Papillomavirus

It is an indication of the ability of a bacterium to produce pathologic changes or disease in the host: A) Virulence B) Pathogenicity C) Attenuation D) In vitro growth rates

B) Pathogenicity

Which of the following neither lay eggs nor deposits larvae in the intestinal canal? A) Hookworm B) Pinworm C) Trichinella spiralis D) Trichuris trichiura

B) Pinworm

Trophozoite that resembles "old man's eyeglasses": A) Plasmodium vivax B) Plasmodium malariae C) Plasmodium falciparum D) Plasmodium ovale

B) Plasmodium malariae

This period is characterized by mild aches, malaise and it follows the period of incubation of the disease: A) Decline phase B) Prodormal stage C) Illness stage D) Convalescence stage

B) Prodormal stage

Microbial decomposition of proteins with the production of H2S and amines is known as: A) Fermentation B) Putrefaction C) Dentrification D) Transpeptidation

B) Putrefaction

Group of microorganisms that can be transmitted to humans by animal vectors like insects: A) Protozoa B) Rickettias C) Chlamydias D) Myoplasmas

B) Rickettias

Which of the following organisms principally infects vascular endothelial cells? A) Salmonella typhi B) Rickettsia typhi C) Haemophilus influenzae D) Coxiella burnetii

B) Rickettsia typhi

The eggs of this organism has a rudimentary lateral spine: A) Schistosoma hematobium C) Schistosoma japonicum B) Schistosoma mansoni D) Fasciola hepatica

B) Schistosoma mansoni

A type of malnutrition associated with inadequate mastication, digestion, absorption, transport and excretion of nutrients: A) Primary malnutrition D) Overnutrition B) Secondary malnutrition E) None C) Undernutrition

B) Secondary malnutrition

This mediates the firm adherence of bacteria to various structures such as skin, heart valves, teeth and even catheters: A) Capsule B) Slime layer C) Pili D) Flagella

B) Slime layer

The following viruses possess RNA polymerase in the virion, except: A) Hepatitis A virus B) Smallpox virus C) Mumps virus D) Rotavirus

B) Smallpox virus

Vancomycin is produced naturally from: A) Strep. nodosus B) Strep. orientalis C) Strep. griseus D) Strep. orchidaceous

B) Strep. orientalis

Nystatin is produced naturally from: A) Strep. nodosus B) Strep.noursei C) Strep. griseus D) Strep. natalensis

B) Strep.noursei

A group B Streptococcus that causes neonatal meningitis and sepsis. A) Staphylococcus aureus B) Streptococcus pyogenes C) Streptococcus pneumoniae D) Staphylococcus epidermidis E) Streptococcus agalactiae

B) Streptococcus pyogenes

Acute glomerulonephritis is a nonsuppurative complication that follows infection by which of the following organisms? A) Streptococcus faecalis B) Streptococcus pyogenes C) Streptococcus pneumoniae D) Streptococcus agalactiae

B) Streptococcus pyogenes

The causative agent of suppurative diseases like pharyngitis and cellulites and nonsuppurative diseases like rheumatic fever and acute glomerulonephritis. A) Staphylococcus aureus B) Streptococcus pyogenes C) Streptococcus pneumoniae D) Staphylococcus epidermidis E) Streptococcus agalactiae

B) Streptococcus pyogenes

Inhibits the release of inhibitory neurotransmitters causing muscle spasms A) Diphtheria toxin D) Toxic shock syndrome toxin B) Tetanus toxin E) Cholera toxin C) Botulinum toxin

B) Tetanus toxin

Which of the following statements is true about whooping cough? A) Infants under the age of 3 months are not susceptible. B) The causative organism adheres to ciliated epithelial cells of the upper respiratory tract. C) Immunity is conferred by killer T cells that recognize cell wall M antigen. D) All of the above

B) The causative organism adheres to ciliated epithelial cells of the upper respiratory tract.

Which of the following is a characteristic of the "positive strand" RNA virus? A) The polymeranse contained in the virion is necessary in the replication. B) The virion RNA can act as its own messenger RNA. C) The virion RNA cannot be extracted in an infectious form. D) Viral messenger RNA is complementary to the virion RNA.

B) The virion RNA can act as its own messenger RNA.

The following statements about measles virus and rubella virus are correct, except: A) They are enveloped RNA viruses. B) Their virions contain RNA polymerase. C) They each have a single antigenic type. D) They are transmitted by respiratory aerosol.

B) Their virions contain RNA polymerase.

Specific process of reproduction among prokaryotes: A) Sporulation B) Transverse binary fission C) Germination D) None of the above

B) Transverse binary fission

A viviparous organism: A) Ascaris lumbricoides B) Trichenella spiralis C) Hookworm D) Pinworm

B) Trichenella spiralis

The adult form of this organism has a whip-like anterior portion: A) Enterobius vermicularis B) Trichuris trichiura C) Ascaris lumbricoides D) Trichinella spiralis

B) Trichuris trichiura

The growing stage of a protozoan parasite is: A) Sporozoite B) Trophozoite C) Merozoite D) Cysts

B) Trophozoite

The causative agent of Varicella (chicken pox) in children and Zoster (shingles) in adults. A) Herpes Simplex virus Type 1 D) Cytomegalovirus B) Varicella-Zoster virus E) Epstein-Barr virus C) Herpes simplex virus Type 2

B) Varicella-Zoster virus

An employee liability law that provides financial supports for workers unemployed because of work related injuries: A) OSH Act B) Workers Compensation Law C) Civilian Rehabilitation D) Coal Mine Health and Safety Act E) None of the above

B) Workers Compensation Law

Blastocyctis hominis is a: A) Zooparasite B) Yeast resembling a protozoan cyst C) New name for Enatamoeba coli D) Bacteria

B) Yeast resembling a protozoan cyst

Ceftriaxone is the most effective drug used to treat this facultative, non-lactose fermenting, gram negative rod which is capable of producing H2S. A. Escherichia coli B. Salmonella typhi C. Shigella dysenteriae D. Vibrio cholerae E. Camphylobacter jejuni

B. Salmonella typhi

A common sampling plan that uses master tables to interpret the results: A) 100% inspection B) Square root system C) Military standard D) A and C

C

A date limiting the time during which a preparation may be expected to have retained its labeled potency: A) Shelf life B) Audit time C) Expiry date D) Any of the above

C

A parenteral is declared to be pyrogenic if the total rise in temperature of the rabbits is: A) Less than 3.3oC in the 8 rabbits C) More than 3.3oC in the 8 rabbits B) More than 3.4oC in the 8 rabbits D) B and C

C

A process wherein the sample is made to liberate iodine, which is then titrated with a standard solution of sodium thiosulfate: A) Volumetric titration B) Back titration C) Iodometry D) Iodimetry

C

A sample of potassium dichromate (K2Cr2O7) weighing 1.5650g was assayed iodometrically using 26.6mL of 0.1120N sodium thiosulfate. Each mL of 0.1N sodium thiosulfate is equivalent to 49.03mg of K2Cr2O7. The percentage purity of the sample is: A) 93.50 B) 95.33 C) 93.33 D) 93.40

C

A tool for detecting variations in a process: A) Pie chart B) Bar chart C) Quality control chart D) T-chart

C

A water content determination method that uses Xylene tube is: A) Gravimetry B) Karl Fischer titrimetry C) Azeotropic method D) Dew point method

C

Accelerated stability testing is, except: A) Performed at higher temperatures to intensify degradation through time B) Done to determine the shelf life of the product in a short period of time C) Enough proof of shelf life and need not be further validated D) The initial method used to determine a product's shelf life

C

Acetylation method of analysis in volatile oils is done to determine: A) Phenol B) Aldehyde C) Alcohol D) Ketone

C

Adsorption indicators are used in: A) Compleximetry B) Acidimetry C) Volumetric precipitation D) Alkalimetry

C

An analyst who determines the strength, potency and percentage purity of a drug or pharmaceutical product is performing: A) Special method B) Identification test C) Pharmaceutical assaying D) Pharmacopoeial test

C

Assay of NaCl in table salt by precipitation as AgCl, filtration, drying and weighing the residue is classified as: A) Direct precipitimetry B) Volhard's method C) Gravimetry D) Residual precipitimetry

C

Calculate the amount of caffeine extracted from coffee beans using 1.0215g of the sample. The volume of 0.0252N H2SO4 added to the extract was 25.4 mL, the excess titrated by 21.75 mL of 0.02115N NaOH. Each mL of 0.02 N H2SO4 is equivalent to 3.8858 mg of caffeine (C9H10O2). The percentage of caffeine is: A) 3.40 B) 3.50 C) 3.42 D) 3.60

C

Caps, bottles, labels and shipping containers are considered as: A) Active components B) Drug products C) Packaging materials D) None of the above

C

Chemical analysis, which determines the amount or percentage content of a certain component in a sample: A) Qualitative B) Proximate C) Quantitative D) Gravimetric

C

Chemical reactions that involves a change in the valence number of reacting substances: A) Neutralization B) Complexation C) Oxidation-reduction D) Precipitation

C

Color of phenolphthalein in vinegar: A) Pink B) Yellow C) Colorless D) Orange]

C

Compute for the percentage loss on ignition of magnesium sulfate hydrated using 3.20 g sample yielding a residue of 2.15g. The loss is: A) 33.81% B) 32.98% C) 32.81 D) 32.75%

C

Disintegration test is done at this temperature: A) 36.5 - 37.5 oC B) 36 - 37 oC C) 35 - 39 oC D) 37 oC

C

Ferric alum TS is used as indicator in volumetric precipitation method if the titrant is: A) AgNO3 B) BaCl2 C) NH4SCN D) NH4Cl

C

How many grams of drug are required to make 500 mL of 25.0% solution? A) 30.0 g B) 50.0 g C) 125 g D) 130 g

C

In column chromatography, the separation of the sample mixture into a series of narrow bands in a column is: A) Chromatogram B) Eluant C) Development D) A and C

C

In gas chromatography, the result in the chromatogram is in terms of: A) Retention on the column C) A and B B) Retention of time/volume D) Volume of sample injected in the column

C

In normal phase chromatography: A) Stationary phase and mobile phase are both polar B) Stationary phase is nonpolar, mobile phase is polar C) Stationary phase is polar, mobile phase is nonpolar D) Stationary phase and mobile phase are both nonpolar

C

In paper chromatography, the data needed to compute for Rf value: A) Distance traveled by the solute C) A and B B) Distance traveled by the solvent D) Distance traveled by the blank

C

In using spectrophotometer to measure the concentration of a sample, the following data were obtained: absorbance of the standard solution = 0.39; absorbance of the sample solution = 0.42; concentration of the standard = 15mcg. The concentration of the sample is: A) 16.15mcg B) 17.5mcg C) 15.75mcg D) 16.0mcg

C

Materials to be sampled include the following, except: A) Intermediate products B) Raw materials C) Records D) Final products

C

Nuclear Magnetic Resonance (NMR) is used for: A) Radioisotopes B) Irradiation C) Identification of chemicals D) A and B

C

Oils with iodine value above 120 are classified as: A) Non-drying B) Semi-drying C) Drying D) None of the above

C

Precipitation method using potassium chromate test solution as indicator and silver nitrate standard solution: A) Volhard B) Fajan C) Mohr D) None of the above

C

Residual titration method under EDTA titrations is applied to metal ion: A) Zinc B) Calcium C) Bismuth D) A and C

C

Safety or toxicity test for infusion plastic sets is conducted using: A) Rabbits B) Dogs C) White mice D) Guinea pigs

C

Substances that have the power of rotating the plane polarized light are said to be: A) Light sensitive B) Active constituents C) Optically active D) Dextrorotatory

C

The Kjeldahl method of analysis is used to determine: A) Fats B) Sugars C) Nitrogen D) Oxygen in organic compounds

C

The acid used in sulfated ash determination of a crude drug is: A) Diluted HCl B) Nitric acid C) Sulfuric acid D) Any of the above

C

The adsorbent in the thin layer chromatography may be as follows, except: A) Alumina B) Silica gel C) Calcium oxide D) Silica gel G

C

The alcohol content of a volatile oil can be determined by: A) Bisulfite method B) Complexation process C) Acetylization process D) A and C

C

The ash content of an organic compound is an impurity of: A) Carbon B) Oil C) Inorganic matter D) Volatile component

C

The ester value determination of substances is applicable to the following, except: A) Fats B) Volatile oils C) Alcohols D) Fatty oils

C

The expiration date is: A) The direct application and interpretation of the knowledge gained from the stability testing. B) Used to limit the period during which a preparation may be expected to have its labeled potency, provided it has been stored as directed. C) A and B are correct D) A and B are incorrect

C

The following are characteristics of a product with quality, except: A) Safe B) Acceptable C) Cheap price D) Effective therapeutically

C

The following are indicators used for acid-base titrations, except: A) Methyl orange B) Phenolphthalein C) Ferric alum D) Methyl red

C

The following are methods of determining the end point in precipitimetry, except: A) Use of adsorption indicators C) Formation of turbidity B) Cessation of precipitation D) Appearance of turbidity

C

The following are optical methods of analysis, except: A) Colorimetry B) Nephelometry C) Chromatography D) Spectrophotometry

C

The following are the basic principles behind chromatographic separation, except: A) Partition B) Adsorption C) Absorption D) Ion-exchange

C

The following compounds are assayed by acidimetrically, except: A) Sodium hydroxide B) Caffeine C) Citric acid D) Zinc oxide

C

The following conditions cause a reaction to go to completion, except: A) Formation of a gas C) Formation of soluble products B) Formation of a precipitate D) Formation of slightly ionized substances

C

The following data were obtained after the moisture and total ash content determination of acacia powder: Wt. of empty crucible 52.452 g Wt. of crucible & sample 61.648 g Wt. of crucible & sample after drying to constant wt. 60.502 g Wt. of crucible and residue left after incineration 53.006g Total ash content in grams: A) 0.545 B) 0.555 C) 0.554 D) 0.445

C

The functions of Quality Control include the following, except: A) Analytical control B) Inspection control C) Auditing D) A and B

C

The gram equivalent of Magnesium oxide (MW = 40.30) is: A) 0.0403 B) 0.0215 C) 21.15 D) 0.4030

C

The heart of HPLC and GC: A) Pumps B) Injection valve C) Column D) Detectors E) Integrator

C

The index of refraction of a volatile oil can be determined by: A) Spectrophotometer B) Pycnometer C) Refractometer D) Polarimeter

C

The indicator used in permanganate titration: A) Methyl orange B) Phenolphthalein C) Permanganate solution D) Methyl red

C

The law related to spectrometry wherein the power of transmitted light decreases exponentially as the thickness of the solution increases arithmetically is: A) Charle's B) Beer's C) Lambert's D) Henry's

C

The method of assay for sulfonamides is: A) Acid-base titration B) Gravimetric C) Diazotization with nitrite D) Redox titration

C

The mobile phase in the gas chromatography is consists of: A) Alcohol-ethyl acetate mixture C) Inert gas B) Alcohol-chloroform mixture D) A and B

C

The most effective way to clean glass apparatus: A) Soaking in detergent C) Treatment with a solution of sodium chromate in sulfuric acid B) Rinsing with nitric acid D) Acetic acid

C

The most widely used chromatographic method in drug analysis is: A) GC B) LLC C) HPLC D) SLC E) TLC

C

The number of mg of KOH needed to neutralize the free acids and saponify the esters in 1g of oil or fat: A) Acid value B) Ester value C) Saponification value D) Iodine value

C

The presence of a cotton fiber in a liquid preparation is considered as a/an: A) Internal defect B) Critical defect C) Ocular defect D) Variable defect

C

The primary standard used to standardize Karl Fischer reagent is: A) Anhydrous sodium carbonate C) Sodium tartrate B) Potassium bipthalate D) Sodium oxalate

C

The term given to the functional group of an organic molecule that absorbs maximum radiation in UV or IR region is: A) Moiety B) Carbonyl C) Chromophore D) Nitrile

C

The type of assay employed in determining the % ZnO in a sample of Zinc White Powder using EDTA as titrant is: A) Volhard's method B) Direct precipitimetry C) Direct compleximetry D) Residual Compleximetry

C

The type of chromatography where the cellulose of the filter paper is used as the adsorbent: A) Column B) Gas C) Paper D) TLC

C

The type of ligand where EDTA belongs; A) Unidentate B) Bidentate C) Hexadentate D) Tridentate

C

The unit of wavelength in spectrometry, except: A) Nanometer B) Micron C) Millimeter D) Micrometer

C

This is the weight of the substance chemically equivalent to 1 mL of the standard solution: A) Titrand B) Equivalence point C) Titer D) End point

C

This is used for quantitative preparation of a standard solution: A) Pipet B) Graduated cylinder C) Volumetric flask D) Buret

C

To find the milliequivalent factor of a reducing agent, divide its molecular weight by: A) No. of electrons gained B) Valence C) No. of electrons lost D) Replace

C

To remove stains of iodine, the most effective chemical substance is: A) Calamansi juice B) Hydrogen peroxide C) Sodium thiosulfate D) Sodium hypochlorite

C

Total quality means: A) Production is responsible for quality C) Quality is everybody's business B) Quality combines strict adherence to standard D) All of the above

C

Two substances reacting upon reaching the end point must have the same: A) Normality B) Volume C) No. of equivalents D) Weight

C

Way to obtain the gram equivalent of a substance (GEW) is: A) Based on the replaceable H+ ion C) A or B B) Based on the valence of the cation D) Based on atomic weight

C

Ways to prepare the sample for Infrared determination, except: A) Use of Mull technique C) Use of alcohol as solvent B) Use of potassium bromide pellet D) Use of liquid petrolatum as solvent

C

The group A streptococci serotype most commonly associated with acute glomerulonephritis: A) 2 B) 7 C) 12 D) 19

C) 12

Leprosy may manifest itself as: A) Lepromatous leprosy B) Tuberculoid leprosy C) A and B D) B and C

C) A and B

Thioglycollate agar is an example of: A) Enriched media B) Selective media C) Anaerobic media D) Differential media

C) Anaerobic media

The most important mechanism of transmission of Pasteurella multicoda from animals to humans: A) Blood-sucking arthropods B) Contact with contaminated feces C) Animal bites and scratches D) Contact with contaminated urine E) Handling of infected tissues

C) Animal bites and scratches

Administration of a toxoid confers: A) Naturally acquired active immunity B) Naturally acquired passive immunity C) Artificially acquired active immunity D) Artificially acquired passive immunity

C) Artificially acquired active immunity

The presence of mammillated eggs or the large adult worm in the stool identifies this organism: A) Trichuris trichiura B) Enterobius vermicularis C) Ascaris lumbricoides D) Strongyloides stercoralis

C) Ascaris lumbricoides

Large, spore-forming rod, whose capsule is composed of poly-D-glutamate and may cause Wool Sorter's disease. A) Meningococcus D) Clostridium tetani B) Gonococcus E) Listeria monocytogenes C) Bacillus antracis

C) Bacillus antracis

Five hours after eating fried rice at a restaurant, you and your friends developed nausea, vomiting and diarrhea. Which of the following organisms is most likely to be the causative agent? A) Clostridium perfringens C) Bacillus cereus B) Enterotoxigenic Escherichia coli D) Salmonella typhi

C) Bacillus cereus

Polymixin is produced from: A) Bacillus subtilis B) Bacillus cereus C) Bacillus polymyxa D) Bacillus anthracis

C) Bacillus polymyxa

A biological sterilization indicator used to test autoclave efficiency: A) Virus B) Bacillus thuringiensis C) Bacillus stearothermophilus D) Clostridium botulinum

C) Bacillus stearothermophilus

Which of the following organisms is a predatory bacterium? A) Salmonella thypinurium B) Pseudomonas aeruginosa C) Bdellovibrio bacteriovirus D) Corynebacterium diphtheriae

C) Bdellovibrio bacteriovirus

Spirochete, which is the causative agent of Lyme disease and can be treated by Doxycycline for early stage and Pen G for late stages. A) Mycobacterium leprae D) Treponema pallidum B) Mycobactrium tuberculosis E) Leptospira interrogans C) Borrelia burgdorferi

C) Borrelia burgdorferi

Regarding the prevention of bacterial diseases by vaccines, which one of the following is least accurate? A) Tetanus toxoid is produced by treating tetanus toxin with formalin, which inactivates its ability to cause disease but leaves its antigenicity intact. B) Diphtheria vaccine contains diphtheria toxoid and produces few side effects when given to children. C) Both the pertussis vaccine and Haemophilus influenza vaccine contain inactivated whole bacteria and produce significant side effects in children. D) The pneumococcal vaccine contains the capsular polysaccharide of many serotypes and is recommended primarily for older people.

C) Both the pertussis vaccine and Haemophilus influenza vaccine contain inactivated whole bacteria and produce significant side effects in children.

Causes paralysis by blocking release of acetylcholine A) Diphtheria toxin D) Toxic shock syndrome toxin B) Tetanus toxin E) Cholera toxin C) Botulinum toxin

C) Botulinum toxin

The following structures normally contain indigenous microbial flora, except: A) Teeth B) Urethra C) Bronchi D) Skin

C) Bronchi

Which of the following zoonotic diseases has no arthropod vector? A) Plague B) Lyme disease C) Brucellosis D) Epidemic typhus

C) Brucellosis

Certain bacterial strains are resistant to the bactericidal activity of the Penicillins and the Cephalosporins, although the growth of the organism is inhibited. This phenomenon (tolerance) is related to a defect or a deficiency of: A) Plasmid that codes for penicillinase B) Autolytic enzymes C) Cellular growth rates D) Certain cytochromes

C) Cellular growth rates

The causative agent of food poisoning from canned good foods. A) Clostridium perfringens D) Corynebacterium diphtheriae B) Clostridium tetani E) Clostridium dificile C) Clostridium botulinum

C) Clostridium botulinum

The single differentiation test between Staphylococcus epidermidis and Staphylococcus aureus: A) Gram staining B) Catalase test C) Coagulase test D) Hemolysis test

C) Coagulase test

The DTP vaccine is composed of which of the following: A) Diphtheria toxin, tetanus toxin, heat-killed Bordetella pertussis B) Diphtheria toxoid, tetanus toxoid, heat-killed Pseudomonas aeruginosa C) Diphtheria toxoid, tetanus toxoid, heat-killed Bordetella pertussis D) Diphtheria toxin, tetanus toxin, heat-killed Pseudomonas aeruginosa

C) Diphtheria toxoid, tetanus toxoid, heat-killed Bordetella pertussis

The substance responsible for the heat resistance of endospores: A) Polymerized d-glutamic acid B) Flagellin C) Dipicolinic acid D) Techoic acid

C) Dipicolinic acid

The dog tapeworm that can be accidentally transmitted to humans: A) Hymenolepsis nana B) Hymenolepsis diminuta C) Dipylidium caninum D) Taenia solium

C) Dipylidium caninum

The eggs of this worm are operculated: A) Taenia solium B) Taenia saginata C) Dipyllobotrium latum D) Dipylidium caninum

C) Dipyllobotrium latum

The fish tapeworm that can be acquired by eating insufficiently cooked, infected fish: A) Taenia solium B) Taenia saginata C) Dipyllobotrium latum D) Dipylidium caninum E) Echinococcus granulosus

C) Dipyllobotrium latum

Visceral larva migrans is acquired by ingesting eggs of a nematode parasite of: A) Pig B) Monkey C) Dog D) Cat

C) Dog

Actively motile trophozoite with directional motility and contains ingested red blood cells: A) Entamoeba coli B) Endolimax nana C) Entamoeba histolytica D) Iodamoeba butschlii E) Dientamoeba fragilis

C) Entamoeba histolytica

Cysts of this organism contains 1-4 nuclei: A) Entamoeba coli B) Endolimax nana C) Entamoeba histolytica D) Iodamoeba butschlii E) Dientamoeba fragilis

C) Entamoeba histolytica

The chromatoid bodies of this organism appear as cigar-shaped or sausage-shaped: A) Entamoeba coli B) Endolimax nana C) Entamoeba histolytica D) Iodamoeba butschlii E) Dientamoeba fragilis

C) Entamoeba histolytica

The findings of ingested red blood cells in a trophozoite practically identifies: A) Entamoeba coli B) Endolimax nana C) Entamoeba histolytica D) Iodamoeba butschlii E) Dientamoeba fragilis

C) Entamoeba histolytica

Which of the following statements is the most important function of antibody in host defenses against bacteria? A) Activation of the lysozyme that degrades the cell wall B) Acceleration of proteolysis of exotoxins C) Facilitation of phagocytosis D) Inhibition of bacterial protein synthesis

C) Facilitation of phagocytosis

The irreversible stage of alcohol damage characterized by liver enlargement: A) Cirrhosis B) Alcoholic hepatitis C) Fatty liver D) All E) None

C) Fatty liver

Toxoplasmosis can be diagnosed by: A) Stool examination B) Thin blood film C) Flourescent antibody tests D) Intradermal test

C) Flourescent antibody tests

The time required for a cell to divide or a population to double is known as: A) Thermal death time B) Growth rate constant C) Generation time D) Thermal death point

C) Generation time

Which of the following does not influence the virulence of Neisseria gonorrhea? A) Polysaccharide capsule B) Lipopolysaccharide endotoxin C) Genus-specific protein D) Production of IgA protease

C) Genus-specific protein

A process by which bacterial endospore returns to its vegetative state: A) Polymorphism B) Mutagenicity C) Germination D) Sporulation

C) Germination

A short-term expression of alcohol toxicity: A) Cirrhosis B) Alcoholism C) Hang-over D) A and B E) B and C

C) Hang-over

The causative agent of herpes genitalis, which can then be treated with Acyclovir. A) Herpes Simplex virus Type 1 D) Cytomegalovirus B) Varicella-Zoster virus E) Epstein-Barr virus C) Herpes simplex virus Type 2

C) Herpes simplex virus Type 2

It is implicated as the cause of carcinoma of the cervix. A) Hepatitis C virus D) Dengue virus B) Cytomegalovirus E) St. Louis encephalitis virus C) Human papilloma virus

C) Human papilloma virus

In the syndrome of post-streptococcal glomerulonephritis: A) Streptococcal nucleases and streptolysin accumulate in the glomerular basement membrane. B) Streptococcal capsular antigen (hyaluronic acid) and glucoronic acid subunits precipitate with antibody and are deposited in the glomeruli in lumpy patterns. C) Immunoglobulin and complement localize in the glomerular basement membrane. D) Hematuria is due to the action of streptolysin O.

C) Immunoglobulin and complement localize in the glomerular basement membrane.

It refers to the invasion or colonization of the body by pathogenic bacteria: A) Disease B) Infestation C) Infection D) Syndromes

C) Infection

The following statements about Hepatitis A are correct, except: A) The initial site of viral replication is the gastrointestinal tract. B) Hepatitis A virus commonly causes asymptomatic infection in children. C) Isolating the virus in a cell culture usually makes diagnosis of hepatitis A. D) Gamma globulin is used to prevent the disease in exposed persons.

C) Isolating the virus in a cell culture usually makes diagnosis of hepatitis A.

The stage in the growth cycle of bacteria that reflects the period during which vigorous metabolic activity occurs but cells do not divide is the: A) Stationary phase B) Log phase C) Lag phase D) Death phase

C) Lag phase

In group A beta hemolytic streptococci, the types are determined by the antigenic specificity of: A) Capsule B) Mucopeptide layer C) M and/or T proteins D) Extracellular products such as streptolysin O, which is produced only by group A streptococci

C) M and/or T proteins

Which of the following statements is true about drug-resistant plasmids? A) Found only in gram-negative bacteria B) Usually confer resistance to a single antibiotic C) May be divided into two distinct genetic components: the RTF and r determinant D) Two of the above

C) May be divided into two distinct genetic components: the RTF and r determinant

This is a saclike invagination of the cytoplasmic membrane that is associated with the DNA of bacterial cells. It functions as the origin of the transverse septum that divides the cell and the binding site of the DNA. A) Nucleoid B) Plasmids C) Mesosomes D) Transposons

C) Mesosomes

Anaerobic bacteria that derive energy by converting formates, acetates and other compounds to methane: A) Microaerophilic B) Metanochromic C) Methanogenic D) Cyanobactreria

C) Methanogenic

The following clinical syndromes are associated with infection by picornavirus, except: A) Myocarditis/Pericarditis B) Hepatitis C) Mononucleosis D) Meningitis

C) Mononucleosis

Which of the following organisms is most likely to be the cause of pneumonia in an immunocompetent patient? A) Nocardia asteroides B) Serratia marcescens C) Mycoplasma pneumoniae D) Legionella pneumophila

C) Mycoplasma pneumoniae

A patient has subacute bacterial endocarditis, which is caused by a member of the viridans group of Streptococcus. Which one of the following sites is most likely to be the source of the organism? A) Skin B) Colon C) Oropharynx D) Urethra

C) Oropharynx

Porins are: A) Cytoplasmic membrane proteins B) Periplasmic proteins C) Outer membrane proteins D) Inclusion bodies

C) Outer membrane proteins

The number of cases of Filipinos with AIDS has increased tremendously over the past few years. This is described as: A) Endemic B) Epidemic C) Pandemic D) Acute

C) Pandemic

This is also known as the Oriental ling fluke: A) Fasciola hepatica B) Fasciolopsis buski C) Paragonimus weatermani D) Chlonorchis sinensis

C) Paragonimus weatermani

Penicillin is a very effective antibacterial drug but their use is limited by allergic reactions. In these allergies, Penicillin acts as a hapten. Which of the following is the most accurate? A) Penicillin is a T-dependent antigens, which bind to receptors on B cells and stimulate an antibody response. B) Penicillin interacts with T cell receptors on CD4-positive T cells and activates them. C) Penicillin binds to carrier proteins, then interacts with the B cell receptor and carrier proteins. The carrier protein epitope is presented to the helper-T cell. D) Penicillin interacts with the early complements (C1, C4, C2 and C3) to release inflammatory mediators.

C) Penicillin binds to carrier proteins, then interacts with the B cell receptor and carrier proteins. The carrier protein epitope is presented to the helper-T cell.

A type of industrial hazard, which includes ambient heat, burn, noise and vibration: A) Biological B) Chemical C) Physical D) Ergonomic E) None of the above

C) Physical

The reservoir host of Balantidium coli infection: A) Dog B) Cat C) Pig D) Cattle

C) Pig

An axostyle is present in this organism: A) Plasmodium vivax B) Plasmodium malariae C) Plasmodium falciparum D) Plasmodium ovale

C) Plasmodium falciparum

As a rule, only rings and crescent-shaped gametocytes are observed in peripheral blood, which includes: A) Plasmodium vivax B) Plasmodium malariae C) Plasmodium falciparum D) Plasmodium ovale

C) Plasmodium falciparum

A phenomenon wherein an organism exhibits plasticity: A) Dimorphism B) Polymorphism C) Pleomorphism D) Fleximorphism

C) Pleomorphism

Human Tcell leukemia virus A) DNA enveloped virus D) RNA non-enveloped virus B) DNA non-enveloped virus E) Viriod C) RNA enveloped virus

C) RNA enveloped virus

A priority program of DOH, which aims at promoting availability of quality services in health centers and hospitals: A) Health Sector Reform Agenda B) National Health Objectives C) Sentrong Sigla D) Health Passport Initiative E) None of the above

C) Sentrong Sigla

The coagulase test, wherein the bacteria causes plasma to clot is used to distinguish: A) Streptococcus pyogenes from Streptococcus faecalis B) Streptococcus pyogenes from Staphylococcus aureus C) Staphylococcus aureus from Staphylococcus epidermidis D) Staphylococcus epidermidis from Neisseria meningitidis

C) Staphylococcus aureus from Staphylococcus epidermidis

Phase in the bacterial growth curve wherein the culture is in the period of balanced growth: A) Log phase B) Lag phase C) Stationary phase D) Death phase

C) Stationary phase

The causative agent of pneumonia and meningitis in adults, and otitis media and sinusitis in children. A) Staphylococcus aureus B) Streptococcus pyogenes C) Streptococcus pneumoniae D) Staphylococcus epidermidis E) Streptococcus agalactiae

C) Streptococcus pneumoniae

It is the relationship between the host and the normal microbiota: A) Disease B) Infection C) Symbiosis D) Parasitism

C) Symbiosis

The following statements about poliovirus and rhinovirus are correct, except: A) They are non-enveloped RNA viruses. B) They each have multiple antigenic types. C) Their virions contain RNA polymerase. D) They do not integrate their genome into the DNA of the host cell.

C) Their virions contain RNA polymerase.

The following statements concerning endotoxins are correct, except: A) They are less potent (ie., less active on weight basis) than exotoxins. B) They are more heat stable than exotoxins. C) They bind to specific cell receptors, whereas exotoxins do not. D) They are part of the bacterial cell wall, whereas exotoxins are not.

C) They bind to specific cell receptors, whereas exotoxins do not.

The capacity of an organism to produce a toxin is known as: A) Pathogenicity B) Virulence C) Toxigenicity D) Toxicity

C) Toxigenicity

The Streptococcus most commonly associated with subacute bacterial endocarditis: A) Streptococcus pyogenes B) Streptococcus agalactiae C) Viridans streptococci D) Streptococcus pneumoniae

C) Viridans streptococci

Viruses are obligate intracellular parasites. The following statements concerning this fact are correct, except: A) Viruses cannot generate energy outside the cell. B) Viruses cannot synthesize proteins outside the cell. C) Viruses must degrade host cell DNA in order to obtain nucleotides. D) Enveloped viruses require host cell membranes to obtain their envelopes.

C) Viruses must degrade host cell DNA in order to obtain nucleotides.

A vitamin that serves as intracellular antioxidants: A) Vit. A B) Vit. D C) Vit. E D) Vit. K E) None of the above

C) Vit. E

Vitamin B deficiency caused by alcoholism produces a neurological disorder called: A) Steven Johnson's Syndrome D) A and B B) Down Syndrome E) None C) Wernicke-Korsakoff Syndrome

C) Wernicke-Korsakoff Syndrome

The following statements regarding Coccidiodes immitis are correct, except: A) It is a dimorphic fungus that grows as a mold in the soil and as spherules in the body. B) Infection usually results from the inhalation of asexual spores (anthroconida), hence the primary site of infection is the lungs. C) When cultured in the laboratory, the organism forms budding yeasts. D) The most important host defense against this organism is cell-mediated immunity.

C) When cultured in the laboratory, the organism forms budding yeasts

This organism causes tropical elephantiasis: A) Loa loa B) Onchocerca volvulus C) Wucheria bancrofti D) Dracunculus medinensis

C) Wucheria bancrofti

Facultative gram negative rods, non-lactose fermenting and may cause enterocolitis (dysentery). A. Escherichia coli B. Salmonella typhi C. Shigella dysenteriae D. Vibrio cholerae E. Camphylobacter jejuni

C. Shigella dysenteriae

A polarimeter is used whether a substance is: A) Dextrorotatory B) Levorotatory C) Optically active D) All of the above

D

A reaction is led to completion by: A) Formation of ash C) Production of slightly ionized molecules B) Formation of insoluble acid D) All of the above

D

A substance that has a high degree of purity and is used in direct standardization purposes: A) Technical grage B) CP grade C) Any of the answers D) Primary standard

D

A type of quality control chart based on the number of defectives found is: A) Go or no-go B) Variable C) Attribute D) A and C

D

Acid insoluble ash is the part of the total ash, which is insoluble in: A) Dilute nitric acid B) Acetic acid C) Dilute sulfuric acid D) Dilute hydrochloric acid

D

Alkaloidal test solutions include: A) Valser's TS B) Warner's TS C) Mayer's TS D) Any of the above

D

An organic compound that changes from one color to another at a certain pH is called: A) Test solution B) Standard solution C) Buffer D) Indicator

D

Analysis of substances that fluoresce falls under: A) Fluorometry B) Requires UV light C) Absorbance reading is obtained D) A and B

D

Antibiotics are assayed by: A) UV-Vis Spectrometry B) IR C) NMR D) Turbidimetry E) Fluorimetry

D

Assay method that determines the presence of pyrogens in injectables: A) Rabbit test B) Limulus amoebocyte lysate test C) White mice test D) A and B

D

Assay of Sodium Nitrite is an example of: A) Neutralization B) Precipitation C) Complexation D) Redox method

D

Beer's plot in spectrometry is prepared to determine: A) Absorbance of the sample C) Absorbance of the blank B) Wavelength to be used D) Concentration of the sample

D

Beer's plot in spectrometry is prepared to determine: A) Absorbance of the sample C) Blank B) Wavelength to be used D) Concentration of the sample

D

Calculate percentage of Atropine in Belladonna powder using 10.7 g sample and adding 20 mL of 0.0225N H2SO4 and back titrated with 12.5 mL of 0.0295N NaOH. Each mL of 0.02N H2SO4 is equivalent to 5.788g of Belladona alkaloids in terms of Atropine. A) 5.60% B) 0.56% C) 0.59% D) 0.22%

D

Chromatography can be used to separate compounds that are: A) Colored B) Colorless C) Precipitate D) A and B

D

Cross-examination and mix up can happen in the following situations, except: A) Improperly dispensed components B) Wrong labels C) Improper partitioning of production area D) Components are kept in tightly-sealed containers

D

Factors that can be manipulated to accelerate instability: A) Temperature B) Moisture C) Light D) All of the above

D

Flame spectroscopy procedures may be divided into: A) Flame emission B) Atomic absorption C) Nuclear magnetic resonance D) A and B

D

In Azeotropic method for water analysis, the solvent that can be used is/are: A) Water B) Toluene C) Xylene D) B and C

D

In TLC, the separation takes place on a planar surface by: A) Dufferential migration B) Electrostatic force C) Relative solubility D) Capillary action E) Any of the above

D

In infrared spectrometry, water is not used as solvent for the sample because the water will: A) Absorb IR radiation C) Not dissolve the sample B) Dissolve the sodium chloride cell holder D) A and B

D

In spectrometry, when a test solution is prepared and observed identically with a reference standard, they are: A) Isometric substances B) Similar solutions C) Similar preparations D) B and C

D

In the Statistical Quality Control, the chart used to measure the variations in the products inspected in the production: A) Variable chart B) P-chart C) Attribute chart D) B and C

D

In the assay of acetic acid (MW = 60.05), each mL of 1N NaOH is equivalent to: A) 0.60000g of acetic acid C) 0.60050g of acetic acid B) 0.00605g of acetic acid D) 0.06005g of acetic acid

D

In thin layer chromatography, finding the spot of the colorless compound in the chromatogram can be done by: A) Use of sulfuric acid spray to obtain a brown color C) Use of carbon tetrachloride spray B) Use of ultraviolet radiation D) A or B

D

In-process control tablets include the following, except: A) Weight B) Hardness and thickness C) Disintegration D0 Bioavailability

D

In-process specification requires that the percent loss on drying should not exceed 0.65% of the sample weight. Based on the given data below, what is the total amount of moisture? Tare wt. 26.839 g Gross wt. 36.506 g Gross wt. after drying 36.495 g A) 0.237 g B) 0.060 g C) 9.607 g D) 0.011 g

D

It is the number of complete cycles that pass a given point per second: A) Wavelength B) Spectrum C) Radiant energy D) Frequency

D

Karl Fischer electrometric titration is a method used to assay for its: A) Oxygen content B) Nitrogen content C) Carbonate content D) Water content

D

Light scattering method of analysis include/s: A) Flourometry B) Turbidimetry C) Nephelometry D) B and C

D

Materials being controlled by Quality Control, except: A) Active ingredient B) Excipients C) Finished products D) Packages

D

Microbiological assay of antibiotics may be performed by using the following, except: A) Test tube dilution B) Cylinder plate C) Paper disc D) Staining method

D

Quantitative determinations of alkaloids may be done by: A) Volumetric B) Gravimetric C) Spectrophotometric D) Any of the above

D

Region in an electromagnetic spectrum: A) Visible B) Infrared C) Ultraviolet D) All of the above

D

Sources of quality variations are as follows, except: A) Technician B) Inadequate procedure C) Equipment D) General manager

D

The Quality Control Department is important to: A) Ensure uniform production of high quality product B) Safeguard public health C) Ensure that minimum standards of the product comply with the BFAD requirements D) All of the above

D

The apparatus used to wash down drops of standard solution clinging to the tip of the buret is: A) Bobcock bottle B) Cassia flask C) Acetylization flask D) Wash bottle

D

The assay of Menadione Sodium Sulfate (Vit. K) uses this method of analysis: A) Gravimetric method B) Acid base titration C) Precipitation method D) Redox titration

D

The chemical factor used in calculation of percent purity of a substance in volumetric analysis: A) mEq weight B) Molarity C) Titer D) A and C

D

The components of a blank used in the assay of a substance are, except: A) Solvent B) Reagent C) Indicator D) Analyte

D

The concentration of an unknown sample in spectrophotometric procedures can be calculated by: A) Use of Beer's plot C) Use of mathematical formula using a reference standard data B) Use of chemical factor D) A or C

D

The gram equivalent weight of potassium permanganate (MW = 158.03) is: A) 29.60 B) 31.60 C) 158.03 D) 52.66

D

The indicator used in the assay of a weak acid reacting with a strong base is: A) Methyl red TS B) Methyl orange TS C) Eriochrome black D) Phenolphthalein TS

D

The infrared region of the spectrometer used to identify a substance has a wavelength range of: A) 200 - 380 nm B) 380 - 780 nm C) 780 - 3000 nm D) 3 - 15 mm

D

The instrument used to measure sodium and potassium ions quantitatively: A) Colorimeter B) Refractometer C) Spectrophotometer D) Flame photometer

D

The iodine value of oils is a quantitative measure of: A) Phenol content B) Saturated fatty acids C) Unsaponifiable matter D) Unsaturated fatty acids

D

The moisture content of a drug may be: A) Water of hydration B) Water in the absorbed form C) Water of emulsion D) A and B

D

The most preferred medium in dissolution testing is: A) Alcohol B) Acetone C) Ether D) Water E) Benzene

D

The name of originator who introduced the adsorption dyes as an indicator in volumetric precipitation method: A) Volhard B) Charles C) Mohr D) Fajan

D

The number of retention samples collected by the Quality Control Inspector from the packaging line is: A) Two times the number required for testing C) Not less than two labeled containers B) Three labeled containers D) Any of A and C

D

The operating speed for paddle in a dissolution apparatus is: A) 150 rpm B) 100 rpm C) 200 rpm D) 50 rpm

D

The primary standard used in the standardization of potassium permanganate is: A) Hydrogen peroxide B) Sulfuric acid C) Potassium dichromate D) Sodium oxalate

D

The specific method for water content determination official in the USP/NF, except: A) Karl Fischer method B) Azeotropic method C) Gravimetric method D) Dew point process

D

The substance being analyzed is the: A) Titrant B) Indicator C) Salt D) Analyte

D

The substance used to form a film over the precipitated silver chloride particles in Volhard's method is: A) Nitric acid B) Sodium chloride C) Triethanolamine D) Nitrobenzene

D

The type of titration in ultimate assay of alkaloids is: A) Direct B) Residual C) Back D) B and C

D

The valence number of Mn in KMNO4 is: A) +2 B) +1 C) +5 D) +7

D

Thiamine is assayed by: A) Refractometry B) Potentiometry C) Colorimetry D) Fluorimetry

D

This determines the shelf life of a product: A) Sampling inspection program C) Validation program B) All of the answers D) Stability testing program

D

To find the gram equivalent weight of Phosphoric acid, divide its molecular weight by: A) 1 B) 0 C) 2 D) 3

D

Type of alkaloidal assay where the total alkaloids are determined: A) Ultimate B) Specific C) Extraction D) Proximate

D

USP limit for tablet content uniformity: A) 90 - 110% B) 90 - 100% C) 95 - 105% D) 85 - 115%

D

Volatile oils are complex compounds composed of: A) Phenols B) Inorganic salts C) Hydrocarbons D) A and C

D

Way to separate the constituents in gravimetric analysis: A) Neutralization process B) Physical means C) Chemical means D) B and C

D

Which is not considered as a primary packaging component? A) Bottles B) Vials C) Caps D) Packer boxes

D

Which of the following is not a QC function? A) In-process analysis B) Inspection of container C) Labeling and packaging materials D) Inventory control

D

Secondary syphilis is characterized by all of the following, except: A) Cutaneous lesions B) Mucous membrane lesions C) Onset is 4 weeks or more after chancre D) Absence of spirochetes in the lesions

D) Absence of spirochetes in the lesions

In which of the following the eggs of parasitic worms can be found: A) Stool B) Urine C) Sputum D) All of the above E) None of the above

D) All of the above

The effectiveness of a disinfectant is influenced by: A) Population size C) Concentration of the disinfectant B) Duration of exposure D) All of the above

D) All of the above

The intimate living together of members of two different species is known as: A) Mutualism B) Commensalism C) Symbiosis D) All of the above

D) All of the above

Tularemia may occur in: A) Ulderoglandular form B) Oculoglandular form C) Pulmonary form D) All of the above

D) All of the above

Specific nutrients transverse the cytoplasmic membrane of bacteria by: A) Activated transport B) Facilitated diffusion C) Carrier-mediated transport D) Any of the above

D) Any of the above

Bacterial genes responsible for drug resistance are known as: A) F plasmids B) R plasmids C) Transposons D) Both B and C

D) Both B and C

Which of the following best describes the mode of action of endotoxin? A) Degrades lecithin in cell membranes B) Inactivates elongation factor-2 C) Blocks release of acetylcholine D) Causes the release of necrosis factor

D) Causes the release of necrosis factor

The following antibiotics interfere with the synthesis of proteins in microorganisms, except: A) Aminoglycosides B) Tetracyclines C) Lincomycin D) Cephalosporins

D) Cephalosporins

This organism is acquired by eating infected raw fish: A) Fasciola hepatica B) Fasciolopsis buski C) Paragonimus weatermani D) Chlonorchis sinensis

D) Chlonorchis sinensis

The following are milk-borne pathogens, except: A) Mycobaterium bovis B) Salmonella C) Streptococcus D) Clostridium

D) Clostridium

Anaerobic, gram positive with terminal spore affecting neutrotransmitters leading to excitatory neurons that are unopposed and extreme muscle spasm. A) Meningococcus D) Clostridium tetani B) Gonococcus E) Listeria monocytogenes C) Bacillus antracis

D) Clostridium tetani

The following events involve recombination of DNA, except: A) Transduction of a chromosomal gene B) Transposition of a mobile genetic element C) Integration of a temperate bacteriophage D) Conjugation, such as the transfer of a R (resistance) factor

D) Conjugation, such as the transfer of a R (resistance) factor

In stained smear, it is usually seen in Chinese letter arrangement having metachromatic granules and may infect the respiratory system. A) Clostridium perfringens D) Corynebacterium diphtheriae B) Clostridium tetani E) Clostridium dificile C) Clostridium botulinum

D) Corynebacterium diphtheriae

The bacterial toxin that causes cessation of mammalian protein synthesis by inactivating EF-2 of eukaryotic cells is produced by: A) Bacillus anthracis B) Clostridium botulinum C) Clostridium tetani D) Corynebacterium diphtheriae

D) Corynebacterium diphtheriae

The male ascaris is distinguished from the female because its tail is: A) Straight B) Slim C) Blunt D) Curved

D) Curved

This antibiotic has been found to be toxic, so its use has been limited only for the treatment of TB: A) Penicillin B) Bacitracin C) Vancomycin D) Cycloserine

D) Cycloserine

Gangcyclovir is beneficial in treating pneumonia and retinitis, while Acyclovir is ineffective. A) Herpes Simplex virus Type 1 D) Cytomegalovirus B) Varicella-Zoster virus E) Epstein-Barr virus C) Herpes simplex virus Type 2

D) Cytomegalovirus

Bacteria lack all of the following organelles, except: A) Mitochondria B) Nucleus C) Mitotic apparatus D) DNA E) Nuclear membrane

D) DNA

The diagnosis of early syphilis is best made by: A) Inoculating guinea pigs with lesion material and examining blood smears 3 days later B) Culture on chocolate agar and incubated with 10% carbon dioxide C) The VDRL test D) Dark-field microscopy of lesion scrapings if a chancre is present

D) Dark-field microscopy of lesion scrapings if a chancre is present

Congenital rubella can be diagnosed in a week-old infant by: A) Demonstration of maternal IgM antibodies against rubella virus B) Testing for HI antibodies specific for the virus in the infant's serum C) Demonstration of circulating IgG antibodies against rubella virus D) Demonstration of rubella IgM antibodies in the infant

D) Demonstration of rubella IgM antibodies in the infant

It causes hemorrhagic fever that can be life threatening. A) Hepatitis C virus D) Dengue virus B) Cytomegalovirus E) St. Louis encephalitis virus C) Human papilloma virus

D) Dengue virus

A 55-year old man develops dysuria and hematuria. A gram stain of urine sample shows gram-negative rods. Culture of the urine on EMB agar reveals non-lactose fermenting colonies without evidence of swarming motility. Which of the following organisms is most likely to be the causative agent of his urinary tract infection? A) Streptococcus faecalis B) Pseudomonas aeruginosa C) Proteus vulgaris D) Escherichia coli

D) Escherichia coli

The intestinal tract is the only natural habitat of this lactose fermenting, facultative anaerobic, nosocomial pathogen: A) Pseudomonas aeruginosa B) Proteus vulgaris C) Klebsiella pneumoniae D) Escherichia coli

D) Escherichia coli

It is an aging process in the life cycle of lake, pond or slow moving river and stream brought about by the accumulation of nutrients needed to sustain aquatic plants and animals accompanied by an increase in the number of organisms: A) Acidification D) Eutrophication B) Alkalinification E) None C) Nitrification

D) Eutrophication

Which is the best method to sterilize Penicillin solution? A) Autoclaving B) Dry heat (oven) C) Ethylene oxide D) Filtration

D) Filtration

National Center for Disease Prevention and Control belongs to what function cluster: A) Internal Management D) Health Regulation Development B) Health Regulation E) None of the above ) External Affairs

D) Health Regulation Development

A causative agent of peptic ulcer: A) Escherichia coli B) Bordetella pertussi C) Shigella dysenteriae D) Helicobacter pylori

D) Helicobacter pylori

Caesarian section has been found to eliminate neonatal complications due to which of the following viruses? A) Varicella-zoster B) Cytomegalovirus C) Poliovirus D) Herpes simplex virus

D) Herpes simplex virus

Which one of the following statements is the most accurate comparison of human, bacterial and fungal cells? A) Human cells undergo mitosis, whereas neither bacteria nor fungi do. B) Human and fungal cells have similar cell wall, in contrast to bacteria whose cell wall contains peptidoglycan. C) Human and bacterial cells have plasmids, whereas fungal cells do not have. D) Human and fungal cells have similar ribosomes, whereas bacterial ribosomes are different.

D) Human and fungal cells have similar ribosomes, whereas bacterial ribosomes are different.

The following statements concerning Interferon are correct, except: A) Interferon inhibits the growth of both DNA and RNA viruses. B) Interferon is induced by double-stranded DNA. C) Interferon of one species acts more effectively in the cells of that species. D) Interferon acts by preventing viruses from entering the cell.

D) Interferon acts by preventing viruses from entering the cell.

Cysts of this organism are usually without glycogen vacuoles (Iodine cysts): A) Entamoeba coli B) Endolimax nana C) Entamoeba histolytica D) Iodamoeba butschlii E) Dientamoeba fragilis

D) Iodamoeba butschlii

It is the most effective skin antiseptic used in medical practice: A) Chlorine B) Merthiolate C) Hydrogen peroxide D) Iodine

D) Iodine

Bacillus anthracis possesses all of the following characteristic, except: A) It is a large, gram-positive rod B) It is a spore-former. C) It produces a potent exotoxin composed of three antigenically distinct thermolabile proteins. D) It can live inside phagocytic cells.

D) It can live inside phagocytic cells.

The following statements regarding the C3 component of the complement cascade are correct, except: A) It is involved in both the classic and the alternative pathways. B) Its C3a fragment can cause anaphylaxis by releasing histamines from mast cells. C) Its C3b fragment binds to both IgG and surface receptors on neutrophils. D) Its C3b fragment is part of the complex that causes lysis of gram-negative bacteria such as Neiserria.

D) Its C3b fragment is part of the complex that causes lysis of gram-negative bacteria such as Neiserria.

In the laboratory, Neisseria meningitidis may be distinguished from Neisseria gonorrhea by: A) Its characteristic morphology and gram stain B) Its unique requirement for CO2 and ability to grow in Thayer-Martin medium C) The presence of pili D) Its ability to ferment particular carbohydrate

D) Its ability to ferment particular carbohydrate

The following are true for endospores, except: A) Highly resistant to disinfectants B) Can survive for many years, especially in soil C) Produced under condition of nutritional deprivation D) Killed by heating at 100oC

D) Killed by heating at 100oC

The causative agent of pneumonia acquired by inhalation from air-conditioners: A) Chlamydia pneumoniae B) Salmonella typhi C) Pneumocystis carinii D) Legionella pneumophila

D) Legionella pneumophila

The following statements concerning Chlamydia are correct, except: A) Chlamydia is a strict intracellular parasite because it cannot synthesize sufficient ATP. B) Chlamydia possesses both DNA and RNA and is bounded by a cell wall. C) Chlamydia trachomatis has multiple serotypes, whereas C. psittaci has only one serotype. D) Most Chlamydias are transmitted by arthropods.

D) Most Chlamydias are transmitted by arthropods.

The following statements regarding the capsules of bacteria are correct, except: A) Most bacterial capsules are polysaccharides and serve to protect the bacteria by inhibiting phagocytosis. B) Bacterial capsules can vary antigenically, and as a result some bacteria have many serologic types. C) Bacterial capsules can be purified and used in vaccines against certain bacteria, example the Pneumococcus. D) Most gram-positive bacteria have capsules, whereas gram-negative bacteria rarely do

D) Most gram-positive bacteria have capsules, whereas gram-negative bacteria rarely do

The region of the body that contains the largest population of microbial flora: A) Skin B) Colon C) Vagina D) Mouth

D) Mouth

Which of the following strategies is most likely to induce lasting intestinal mucosal immunity to poliovirus? A) Parenteral (intramuscular) administration of inactivated vaccine B) Oral administration of poliovirus immunoglobulin C) Parenteral administration of live vaccine D) Oral administration of live vaccine

D) Oral administration of live vaccine

Neisseria are all: A) Gram positive B) Anaerobes C) Coagulase positive D) Oxidase positive

D) Oxidase positive

A thin proteinaceous appendage necessary for bacterial conjugation: A) Cilium B) Trichome C) Flagellum D) Pilus

D) Pilus

Rotavirus A) DNA enveloped virus D) RNA non-enveloped virus B) DNA non-enveloped virus E) Viriod C) RNA enveloped virus

D) RNA non-enveloped virus

An example of a streptococcal disease sequela: A) Scarlet fever B) Pharyngitis C) Puerperal sepsis D) Rheumatic fever

D) Rheumatic fever

The identification of bacteria by serologic test is based on the presence of specific antigens. Which of the following bacterial components is least likely to contain useful antigens? A) Capsule B) Flagella C) Cell wall D) Ribosomes

D) Ribosomes

This disinfectant is useful in preventing gonococcal ophthalmia neonatorum among infants whose patients have Gonorrhea: A) Merthiolate B) Mercurochrome C) Silver sulfadiazine D) Silver nitrate

D) Silver nitrate

A class in which only parasitic forms occur: A) Sarcodina B) Mastigophora C) Ciliata D) Sporozoa

D) Sporozoa

Norma flora of the skin but may cause "stitch abscess" and sepsis. A) Staphylococcus aureus B) Streptococcus pyogenes C) Streptococcus pneumoniae D) Staphylococcus epidermidis E) Streptococcus agalactiae

D) Staphylococcus epidermidis

The following are communicable diseases, except: A) Measles B) Pneumonia C) Hepatitis D) Tetanus

D) Tetanus

The following are true about prokaryotes, except: A) They are relatively small. B) Their genes are dedicated to essential functions only. C) They lack autonomous organelles. D) Their genetic materials are enclosed in a nuclear membrane.

D) Their genetic materials are enclosed in a nuclear membrane.

The following statements are true about endotoxins, except: A) They are also known as lipopolysaccharides. B) They are integral part of the cell walls of gram-negative bacteria. C) They are normally not as toxic as bacterial exotoxins. D) Their toxicity can be destroyed by autoclaving.

D) Their toxicity can be destroyed by autoclaving.

The following statements about influenza virus and rabies virus are correct, except: A) They are enveloped RNA viruses. B) Their virions contain RNA polymerase. C) Vaccines containing killed organisms are available for both viruses. D) They each have a single antigenic type.

D) They each have a single antigenic type.

Stimulates T cells to produce cytokines A) Diphtheria toxin D) Toxic shock syndrome toxin B) Tetanus toxin E) Cholera toxin C) Botulinum toxin

D) Toxic shock syndrome toxin

Penicillin is effective for this spirochete organism causing syphilis. A) Mycobacterium leprae D) Treponema pallidum B) Mycobactrium tuberculosis E) Leptospira interrogans C) Borrelia burgdorferi

D) Treponema pallidum

Rod-shaped bacteria curved to form commas are known as: A) Bacilli B) Cocci C) Spiral D) Vibrios

D) Vibrios

Backyard gardening, community project and putting of herbal plants are among the major activities under this program: A) Araw ng Sangkap Pinoy D) WHO B) Nutrition Program E) None of the above C) National Drug Policy Program

D) WHO

Its main objective is attainment by all people of the highest possible level of health: A) Philhealth B) DOH Hospitals C) UNICEF D) WHO E) None of the above

D) WHO

A halophilic coma shaped, gram negative bacteria. A. Escherichia coli B. Salmonella typhi C. Shigella dysenteriae D. Vibrio cholerae E. Camphylobacter jejuni

D. Vibrio cholerae

The peptidoglycan of gram-negative bacteria probably is: A) 20 layers thick D) 80 layers thick B) 40 layers thick E) 1 layer thick C) 10 layers thick

E) 1 layer thick

Compounds that combine with atmospheric moisture to produce highly acidic rain, snow, hair or fog: A) Carbon dioxide B) Sulfur dioxide C) Nitrogen oxide D) A and B E) B and C

E) B and C

Filariform larva that can penetrate through the skin: A) Enterobius vermicularis D) A and B B) Strongyloides stercoralis E) B and C C) Hookworm

E) B and C

Which of the following techniques is used in quantitative estimations of infections with parasitic worms? A) Formalin-ether B) Kato thick smear C) Stool dilution D) A and B E) B and C

E) B and C

Stimulates the production of cyclic AMP by adding ADP-ribose to a G protein A) Diphtheria toxin D) Toxic shock syndrome toxin B) Tetanus toxin E) Cholera toxin C) Botulinum toxin

E) Cholera toxin

Anaerobic, gram positive, spore-forming rods, which is a normal flora of the intestine. It has enterotoxin that causes watery diarrhea and may lead to pseudomembranous colitis. A) Clostridium perfringens D) Corynebacterium diphtheriae B) Clostridium tetani E) Clostridium dificile C) Clostridium botulinum

E) Clostridium dificile

The adult form of this tapeworm has only four segments: A) Taenia solium B) Taenia saginata C) Dipyllobotrium latum D) Dipylidium caninum E) Echinococcus granulosus

E) Echinococcus granulosus

The causative agent of Infectious Mononucleosis (IM) and is associated with Burkitt's lymphoma in East African children. No drug is effective to treat the disease. A) Herpes Simplex virus Type 1 D) Cytomegalovirus B) Varicella-Zoster virus E) Epstein-Barr virus C) Herpes simplex virus Type 2

E) Epstein-Barr virus

Spirochete, which is the causative agent of leptospirosis. A) Mycobacterium leprae D) Treponema pallidum B) Mycobactrium tuberculosis E) Leptospira interrogans C) Borrelia burgdorferi

E) Leptospira interrogans

Aerobic, non-spore forming organism that exhibits tumbling motility and capable of causing meningitis and sepsis in newborn and immunocompromised patients. A) Meningococcus D) Clostridium tetani B) Gonococcus E) Listeria monocytogenes C) Bacillus antracis

E) Listeria monocytogenes

The following are attributed to fetal alcohol syndrome, except: A) Smaller size B) Deformities of limb C) Heart defects D) Poor coordination E) None

E) None

A lung disease caused by cotton-mill dust: A) Mesothelioma B) Chemical hazards C) Physical hazards ` D) Ergonomic hazards E) None of the above

E) None of the above

The following are functions of Amiotic fluid, except: A) Allows the movement of fetus B) Protects against mechanical injury C) Provides stable temperature D) Allows the fetus to float E) None of the above

E) None of the above

Wild birds are important reservoirs of this virus. A) Hepatitis C virus D) Dengue virus B) Cytomegalovirus E) St. Louis encephalitis virus C) Human papilloma virus

E) St. Louis encephalitis virus

Finding the rhabditiform larva in the stool identifies this organism: A) Hookworm D) Ascaris lumbricoides B) Whipworm E) Strongyloides stercoralis C) Pinworm

E) Strongyloides stercoralis

A gram negative, coma shaped microaerophilic rod that causes enterocolitis and can be cultured in Skirrow's agar. A. Escherichia coli B. Salmonella typhi C. Shigella dysenteriae D. Vibrio cholerae E. Camphylobacter jejuni

E. Camphylobacter jejuni


Related study sets

Module 11: Wireless Network Security

View Set

EMT Chapter 17-19, 21 & 22 - Neurologic Emergencies, Gastrointestinal and Urologic Emergencies, Endocrine and Hematologic, Toxicology, Psychiatric Emergencies, EMT

View Set

Forensic Analysis/Restriction enzymes

View Set

biochemistry test - unit 2 module 6

View Set

Electric vehicle charging systems midterm

View Set

NC BLET JUVENILE LAWS AND PROCEDURES

View Set